Вы находитесь на странице: 1из 128

Revista da Olimpada Regional de

Matem atica Santa Catarina


N
o
3, 2006
O
L
I
M
P

A
D
A
R
E
G
IONAL D
E
M
A
T
E
M

T
I
C
A
S
A
N
T
A
C
ATARINA
-
U
F
S
C
UNIVERSIDADE FEDERAL DE SANTA CATARINA
Reitor: L ucio Jose Botelho
Vice-Reitor: Ariovaldo Bozan
PR

O-REITORIA DE CULTURA E EXTENS

AO - PRCE
Pr o-Reitora: Eunice Sueli Nodari
DEPARTAMENTO DE APOIO
`
A EXTENS

AO - DAEx
PR

O-REITORIA DE ENSINO DE GRADUAC



AO - PREG
Pr o-Reitor: Marcos Lan
CENTRO DE CI

ENCIAS F

ISICAS E MATEM

ATICAS - CFM
Diretor: Mericles Thadeu Moretti
Vice-Diretor: Tarciso Ant onio Grandi
DEPARTAMENTO DE MATEM

ATICA
Chefe: Nereu Estanislau Burin
Sub-Chefe: Carmem Suzane Comitre Gimenez
Apoio:
INSTITUTO DO MIL

ENIO - Avan co Global e Integrado da Matematica Brasileira


SOCIEDADE BRASILEIRA DE MATEM

ATICA - SBM
CATALOGAC

AO NA PUBLICAC

AO PELA BIBLIOTECA UNIVERSIT

ARIA DA
UNIVERSIDADE FEDERAL DE SANTA CATARINA
Revista da Olimpada Regional de Matem atica Santa Catarina/
Universidade Federal de Santa Catarina. Centro de Ciencias
Fsicas e Matem aticas. n.1 (2004) . Florian opolis:[s.n],
2004 -
v.: 23 cm
Anual
ISSN 1679-7612
1. Matem atica Competi c oes. 2. Matem atica Quest oes, Problemas,
exerccios. I. Universidade Federal de Santa Catarina. II Centro de
Ciencias Fsicas e Matem aticas.
Comissao da Olimpada Regional de Matematica de SC:
Coordenador: Jose Luiz Rosas Pinho.
Professores: Carmem Suzane Comitre Gimenez, Eliezer Batista, Licio Hernanes Bezerra,
Nereu Estanislau Burin, Waldir Quandt e William Glenn Whitley.
Bolsistas da olimpada: Ana Paula Bertoldi Oberziner, Rodrigo Maciel Rosa, Tatiana
Sprandel.
Bolsistas do PET - Matematica: Ana Beatriz Michels, Carla M orschb acher, Caue Ro-
ratto, Cinthia Marques Vieira Andretti, Felipe Vieira, Graciele Amorim, Helosa Cristina da
Silva, Leonardo Koller Sacht, Louise Reips, Marcos Teixeira Alves, Monique M uller Lopes
Rocha e Paulo Ricardo Bo.
Comite Editorial da Revista da Olimpada Regional de Matematica Santa Cata-
rina:
Ana Beatriz Michels
Felipe Vieira
Graciele Amorim
Karla Christina da Costa Kagoiki
Jose Luiz Rosas Pinho
Monique M uller Lopes Rocha
Waldir Quandt
William Glenn Whitley
Editora cao Eletr onica: Tiragem:
Alda Dayana Mattos 1000 exemplares
Ana Beatriz Michels
Monique M uller Lopes Rocha
Rodrigo Maciel Rosa
Arte da Capa: Postagem:
Renata Leandro Becker Segundo Semestre de 2005.
Revista da Olimpada Regional de Matematica Santa Catarina N.
o
3, 2006
ISSN 1679-7612
Sumario
Apresenta cao 7
VII ORM (2004) 9
Problemas . . . . . . . . . . . . . . . . . . . . . . . . . . . . . . . . . 11
Nvel 1 . . . . . . . . . . . . . . . . . . . . . . . . . . . . . . . . 11
Nvel 2 . . . . . . . . . . . . . . . . . . . . . . . . . . . . . . . . 13
Nvel 3 . . . . . . . . . . . . . . . . . . . . . . . . . . . . . . . . 15
Solu c oes . . . . . . . . . . . . . . . . . . . . . . . . . . . . . . . . . . 17
Nvel 1 . . . . . . . . . . . . . . . . . . . . . . . . . . . . . . . . 17
Nvel 2 . . . . . . . . . . . . . . . . . . . . . . . . . . . . . . . . 21
Nvel 3 . . . . . . . . . . . . . . . . . . . . . . . . . . . . . . . . 25
Premiados . . . . . . . . . . . . . . . . . . . . . . . . . . . . . . . . . 29
Nvel 1 . . . . . . . . . . . . . . . . . . . . . . . . . . . . . . . . 29
Nvel 2 . . . . . . . . . . . . . . . . . . . . . . . . . . . . . . . . 32
Nvel 3 . . . . . . . . . . . . . . . . . . . . . . . . . . . . . . . . 36
Escolas Participantes . . . . . . . . . . . . . . . . . . . . . . . . . . . 40
Artigo 47
Os N umeros e o Innito
Ivan Pontual Costa e Silva 49
Artigo 63
Problemas Olmpicos: Analise quanto `as Diferentes Tecnicas de
Resolu cao
Juliana Duarte Zacchi 65
Artigo 77
O Problema da Divisao da Pizza
William Glenn Whitley 79
Artigo 85
Se a Terra nao e Plana , quais sao as Rela c oes Metricas adequadas
para determinarmos Comprimentos e

Angulos?
Celso Melchiades Doria 87
Solu c oes de problemas propostos na revista anterior 107
Problemas propostos 117
Outras olimpadas 121
Informa c oes gerais 125
Envio de Problemas e Solu c oes . . . . . . . . . . . . . . . . . . . . . 127
Envio de Artigos . . . . . . . . . . . . . . . . . . . . . . . . . . . . . 127
Cadastramento . . . . . . . . . . . . . . . . . . . . . . . . . . . . . . 127
Como adquirir a revista . . . . . . . . . . . . . . . . . . . . . . . . . 127
Erramos . . . . . . . . . . . . . . . . . . . . . . . . . . . . . . . . . . 128
Fale Conosco . . . . . . . . . . . . . . . . . . . . . . . . . . . . . . . 128
7
Apresenta cao
A Revista da Olimpada Regional de Matem atica (ORM) de Santa Cata-
rina tem por objetivo divulgar esta Olimpada e a Olimpada Brasileira de
Matem atica (OBM). A ORM ocorreu em 2004 em sua 7
a
edi c ao contando com
o apoio, externamente, da Sociedade Brasileira de Matem atica (SBM) e do In-
stituto do Milenio - Avan co Global e Integrado da Matem atica Brasileira (IM-
AGIMB), e internamente das Pr o-Reitorias de Cultura e Extens ao (PRCE), de
Assuntos Estudantis (PRAE) e de Ensino e Gradua c ao (PREG) da Universi-
dade Federal de Santa Catarina. Ela e realizada como um projeto de extens ao
do Departamento de Matem atica, com a participa c ao de um grupo de profes-
sores e de alunos com bolsas da PRCE, alunos do PET - Matem atica e alunos
colaboradores, todos do Curso de Matem atica da UFSC.
Neste segundo n umero da Revista apresentamos as provas (com solu c oes) da
IV, V e VI ORM (anos 2001, 2002 e 2003 respectivamente), artigos dos profes-
sores da UFSC Licio Hernanes Bezerra e William Glenn Whitley, participantes
do projeto da ORM, solu c oes de problemas propostos no n umero anterior e
novos problemas propostos. Este n umero foi nanciado atraves do programa
PROEXTENS

AO 2003, do Departamento de Apoio ` a Extens ao (DAEx) da


PRCE da UFSC, e pelo IM - AGIMB, sendo apresentado na 4
a
Semana de
Ensino, Pesquisa e Extens ao (SEPEX) da UFSC em setembro de 2004.
Solicitamos a todos os interessados que nos enviem sugest oes, problemas
e solu c oes de problemas propostos, e que submetam artigos para an alise do
Comite Editorial e publica c ao nesta Revista.
Nossa inten c ao e atingir o maior n umero de escolas em Santa Catarina e,
se possvel, divulgar a ORM por todo o pas. As escolas que n ao tiverem rece-
bido este segundo n umero poder ao solicitar um exemplar entrando em contato
conosco ou consultando nossa p agina. Nossa esperan ca e que, algum dia, todas
as escolas do Estado estejam participando das Olimpadas de Matem atica.
Florian opolis, 14 de outubro de 2004.
Jose Luiz Rosas Pinho
Coordenador das Olimpadas de Matem atica de Santa Catarina
Revista da ORM/SC n
o
3, 2006
8
Revista da ORM/SC n
o
3, 2006
VII ORM (2004)
10 VII ORM (2004)
Revista da ORM/SC n
o
3, 2006
Problemas 11
Problemas
Nvel 1
1. Lal a, Lele e Lili s ao tres amigas que moram juntas em Lobelia. Porem,
Lal a s o ca em Lobelia sexta, s abado e domingo, pois ca em Lupercio
durante o resto da semana. Considerando que um mes tem trinta dias e
quatro nais de semana, responda:
(a) Como deve ser feita a divis ao, entre as tres amigas, das despesas
xas de moradia e alimenta c ao de R$ 840,00, de modo que essas
despesas sejam divididas proporcionalmente ao tempo de permanencia
de cada uma em Lobelia?
(b) Quanto dinheiro sobra para Lal a para outras despesas sabendo-se
que ela ganha R$ 2.004,00 por mes e, alem das despesas em Lobelia,
gasta R$ 1.100,00 sozinha para manter um outro apartamento em
Lupercio?
2. Considere a m aquina de transformar n umeros, que funciona conforme
as seguintes regras:
(a) recebe um n umero e remove seu primeiro algarismo, guardando o
n umero resultante;
(b) pega o resultado de (a), inverte a ordem dos seus algarismos e guarda
o n umero;
(c) soma os algarismos do n umero gerado em (b) e anexa a soma ao
lado direito do n umero;
(d) mostra o resultado nal.
Exemplos: 9987 987 789 78924.
1023 23 32 325.
ALERTA! No sistema posicional de representa c ao de n umeros, n ao se
permite o algarismo 0 no lado esquerdo da representa c ao.
Ache um n umero de quatro algarismos que n ao e alterado por esta m aquina.
H a outro?
Revista da ORM/SC n
o
3, 2006
12 VII ORM (2004)
3. Numa sala h a doze caixas, de mesma aparencia externa, contendo barras
de ouro. Cada barra de ouro pesa 1 kg. Sabe-se que nenhuma caixa est a
vazia. As caixas s ao repartidas em quatro grupos de tres caixas cada e,
ao serem pesados, verica-se que a totalidade de ouro em cada grupo e
de 7 kg. Em seguida, as caixas s ao repartidas em tres grupos de quatro
caixas cada, e, ao serem pesados, verica-se que h a um grupo com 19 kg
de ouro, um com 4 kg e o terceiro com 5 kg. Qual e a maior quantidade
de barras de ouro que est a colocada em uma caixa? Quais s ao as possveis
distribui c oes das barras nas caixas?
4. O Sr. Silva sai de sua cidade natal para trabalhar em Shangril a. Ele chega
em Shangril a no unico navio que serve esta cidade em 3 de agosto de 1845
e l a ca trabalhando ate 2 de agosto de 1846, inclusive. Depois, ele tira
vinte dias de ferias visitando cidades pr oximas a Shangril a. Sabendo que
o navio chega em Shangril a a cada 27 dias de manh a cedo e parte no
mesmo dia a noite, qual ser a a primeira data em que ele poder a voltar
para a sua cidade natal?
5. Considere o ladrilho quadrado, ao lado,
subdividido em regi oes. Apresente uma
maneira de colorir estas regi oes com o
mnimo possvel de cores, de modo que,
ao formar um quadrado com nove desses
ladrilhos, regi oes contguas tenham cores
diferentes.
Observa cao: Se uma regi ao tem apenas
um vertice em contato com um lado ou um
vertice de outra regi ao, ent ao tais regi oes
nao s ao consideradas contguas.
Revista da ORM/SC n
o
3, 2006
Problemas 13
Nvel 2
1. A sociedade kardaliana usava um sistema posicional com apenas quatro
algarismos distintos para representar os n umeros naturais: , , e .
Eles desenvolveram as opera c oes de adi c ao e multiplica c ao e descobriram
que as opera c oes que envolvem somente adi c ao ou multiplica c ao podiam
ser reordenadas ou agrupadas de qualquer modo sem alterar o resultado
(propriedades associativa e comutativa) e que a multiplica c ao distribuia
sobre a adi c ao (x (y + z) = (x y) + (x z)). Os valores b asicos das
opera c oes s ao dados abaixo:
+









(a) Qual e a representa c ao no sistema kardaliano para o nosso 0 e para
o nosso 1 ?
(b) Qual e o primeiro n umero que e representado com dois dgitos? E
com tres dgitos?
(c) Qual e a rela c ao entre os dois n umeros do item (b)?
(d) Represente o valor da seguinte express ao no sistema kardaliano:
( )+ .
2. Considere a m aquina de transformar n umeros, que funciona conforme
as seguintes regras:
(a) recebe um n umero e remove seu primeiro algarismo, guardando o
n umero resultante;
(b) pega o resultado de (a), inverte a ordem dos seus algarismos e guarda
o n umero;
(c) soma os algarismos do n umero gerado em (b) e anexa a soma ao
lado direito do n umero;
(d) mostra o resultado nal.
Revista da ORM/SC n
o
3, 2006
14 VII ORM (2004)
Exemplos: 9987 987 789 78924.
1023 23 32 325.
ALERTA! No sistema posicional de representa c ao de n umeros, n ao se
permite o algarismo 0 no lado esquerdo da representa c ao.
Ache um n umero de cinco algarismos que n ao e alterado por esta m aqui-
na. H a outro?
3. Encontre primos p e q tais que 167p +q = 2004.
4. Numa sala h a doze caixas, de mesma aparencia externa, contendo barras
de ouro. Cada barra de ouro pesa 1kg. Sabe-se que nenhuma caixa est a
vazia. As caixas s ao repartidas em quatro grupos de tres caixas cada e,
ao serem pesados, verica-se que a totalidade de ouro em cada grupo e
de 7kg. Em seguida, as caixas s ao repartidas em tres grupos de quatro
caixas cada, e, ao serem pesados, verica-se que h a um grupo com 18kg
de ouro, um com 4kg e o terceiro com 6kg. Qual e a maior quantidade de
barras de ouro que est a colocada em uma caixa? Quais s ao as possveis
distribui c oes das barras nas caixas?
5. O Sr. Silva saiu de Eldorado e chegou em Shangril a em 3 de agosto de
1845 no navio Estrela do Leste, o unico que serve Shangril a e que por
l a passa a cada vinte e sete dias. O Sr. Silva trabalhou em Shangril a
ate 17 de julho de 1849, inclusive, tirou ferias e viajou de diligencia na
manh a do dia seguinte para a Terra do Sonho. A viagem entre essas duas
cidades dura setenta e duas horas. Sabemos que o navio sempre chega em
Shangril a de manh a cedo e parte no nal da tarde do mesmo dia, e que h a
duas diligencias di arias da Terra do Sonho para Shangril a, uma partindo
` as oito e outra ` as vinte horas. Ap os permanecer dez dias na Terra do
Sonho, em que dia o Sr. Silva deve partir de volta para Shangril a para
pegar o primeiro navio disponvel para voltar a Eldorado, considerando
que ele quer permanecer o maior tempo possvel na Terra do Sonho?
Revista da ORM/SC n
o
3, 2006
Problemas 15
Nvel 3
1. Considere o quadril atero ABCD da gura
em que AB=2, AE=DE=1, BE AD,
EBC = 60
o
e DCB = 60
o
. Qual e a
area do ABCD?
C
D E
A
B
2. A sociedade kardaliana usava um sistema posicional com apenas quatro
algarismos distintos para representar os n umeros naturais: , , e .
Eles desenvolveram as opera c oes de adi c ao e multiplica c ao e descobriram
que as opera c oes que envolvem somente adi c ao ou multiplica c ao podiam
ser reordenadas ou agrupadas de qualquer modo sem alterar o resultado
(propriedades associativa e comutativa) e que a multiplica c ao distribua
sobre a adi c ao (x (y + z) = (x y) + (x z)). Os valores b asicos das
opera c oes s ao dados abaixo:
+









(a) Qual e a representa c ao no sistema kardaliano para o nosso 0 e para
o nosso 1 ?
(b) Qual e o primeiro n umero que e representado com dois dgitos? E
com tres dgitos?
(c) Qual e a rela c ao entre os dois n umeros do item (b)?
(d) Represente o valor das seguintes express oes no sistema kardaliano:
i. + + ;
ii. .
3. Considere a m aquina de transformar n umeros, que funciona conforme
as seguintes regras:
Revista da ORM/SC n
o
3, 2006
16 VII ORM (2004)
(a) recebe um n umero e remove seu primeiro algarismo, guardando o
n umero resultante;
(b) pega o resultado de (a), inverte a ordem dos seus algarismos e guarda
o n umero;
(c) soma os algarismos do n umero gerado em (b) e anexa a soma ao
lado direito do n umero;
(d) mostra o resultado nal.
Exemplos: 9987 987 789 78924.
1023 23 32 325.
ALERTA! No sistema posicional de representa c ao de n umeros, n ao se
permite o algarismo 0 no lado esquerdo da representa c ao.
Quais s ao os n umeros de seis algarismos que n ao s ao alterados pela
m aquina?
4. Considere a fun c ao f(x) =
16
x 1
+
32
x 2
denida nos pontos do intervalo
[0 , 3], exceto nos pontos 1 e 2. Suponha que L e uma lista de n umeros,
em ordem crescente, que come ca em zero, termina em 3 e tal que os pares
de n umeros consecutivos de L dividem o intervalo em quinhentos sub-
intervalos de comprimentos iguais. Qual ponto x de L faz com que f(x)
seja o maior valor possvel?
5. Encontre primos p e q tais que 25p +q = 2004, com 400 q 600.
Voce Sabia? Um n umero e dito regular se sua decomposi c ao em
fatores primos apresenta apenas potencias de 2, 3 e 5. Exemplo: 60
e um n umero regular, pois 60 = 2
2
3 5.
Revista da ORM/SC n
o
3, 2006
Solu c oes 17
Solu c oes
Nvel 1
1. (a) Lal a ca em Lobelia 3 dias por semana, nas quatro semanas (4 ns
de semana) do mes. Portanto Lal a ca 12 dias por mes em Lobelia.
Assim
12
30
das despesas do mes devem ser repartidas entre as tres
amigas, e o resto entre Lele e Lili:
12
30
840 = 336 e 336 3 = 112.
Agora, 840 336 = 504 e 504 2 = 252.
Ainda: 252 + 112 = 364. Portanto Lal a pagar a R$112,00 e Lele e
Lili R$ 364,00 cada uma.
(b) 1100 +112 = 1212. Assim sobram pra Lal a 2004 1212 = 792 reais
por mes.
2. Seja abcd um n umero em que a, b, c, d, s ao algarismos com a ,= 0 (n umero
de 4 algarismos). Ent ao a m aquina faz o seguinte:
abcd bcd dcb dcbx,
em que x deve ser a soma dos algarismos d, c e b. Por outro lado, como
queremos que abcd n ao seja alterado por esta m aquina, teremos duas
possibilidades:
(i) d ,= 0. Neste caso, x deve ser igual a d, ou seja, a soma de d,b, e
c deve ser igual a d. Mas isto implica que a soma de b com c seja
igual a zero, o que e impossvel.
(ii) d = 0 e teremos:
abc0 bc0 cb cbx,
em que x = b+c e esta soma deve ser um n umero de dois algarismos
terminado em zero. A unica possibilidade e x = 10. Portanto temos:
abc0 bc0 cb cb10.
Da conclumos: c = 1, a = c = 1. Como b + c = 10, temos b = 9.
Portanto o n umero (o unico) que n ao se altera pela m aquina e: 1910.
Revista da ORM/SC n
o
3, 2006
18 VII ORM (2004)
3. Pela reparti c ao em quatro grupos de tres caixas, cada um pesando 7 kg,
sabemos que o total de ouro nas caixas e 28 kg (4 7). Como n ao h a
caixas vazias, e como na reparti c ao em grupos de quatro caixas h a um
grupo pesando 4 kg e outro pesando 5 kg, teremos para estes grupos:
4 kg + 5 kg + 19 kg =28
1 2
1 1
1 1
1 1
Portanto, pelo menos 7 caixas possuem exatamente 1 kg de ouro cada.
Olhando a distribui c ao em quatro grupos de tres caixas cada teremos:
7 kg 7 kg 7 kg 7 kg
1 1 1 1
1 1 1 2
5 5 5 4
Observe que esta e a unica distribui c ao possvel das barras de 1 kg nas
caixas dos grupos e que a caixa com 2 kg deve pertencer ao quarto grupo.
Revista da ORM/SC n
o
3, 2006
Solu c oes 19
Portanto 3 caixas devem conter 5 kg de ouro e a ultima caixa do quarto
grupo deve conter 4 kg de ouro. Estas ultimas caixas comp oem o terceiro
grupo de 4 caixas:
5 + 5 + 5 + 4 = 19.
Portanto a maior quantidade de ouro em uma caixa e 5Kg e h a uma unica
distribui c ao possvel.
4. O Sr. Silva trabalhou exatamente 365 dias. Tirou 20 dias de ferias, ou
seja, de 3 de agosto de 1846 a 22 de agosto de 1846 (inclusive). Portanto,
desde sua chegada a Shangril a ate o dia 22 de agosto de 1846 passaram-se
385 (365+20) dias. Mas
385 = 27 14 + 7.
Portanto o ultimo navio passou em Shangril a no dia 16 de agosto de 1846
(de 16 a 22 de Agosto, inclusive s ao 7 dias). O pr oximo navio passar a
daqui a 27 dias. Contando, temos 15 dias em agosto (sem contar o dia
16 ate 31 de agosto). Mais 12 dias em setembro e teremos 27 dias.
Portanto o Sr. Silva poder a voltar para sua cidade natal no dia 12 de
setembro de 1846 (se quiser voltar na primeira data possvel).
5. Com uma cor e obviamente impossvel. Com duas cores tambem e im-
possvel pois h a regi oes contguas com um mesmo lado contguo a duas
outras regi oes:
Vejamos ent ao com 3 cores. Uma possibilidade e (cores 1, 2 e 3):
Revista da ORM/SC n
o
3, 2006
20 VII ORM (2004)
1 2
3
1
2
1 2
3
2
1
2 3
3
3
2
1
3
1
Mas essa possibilidade n ao permite juntar os ladrilhos lado a lado nem
um sobre o outro (mesmo rotacionando-os). Uma solu c ao e:
1
3
3
3
3
2 3
2
1
2
2
3
1
2
1 2
1
1
Nessa solu c ao os ladrilhos devem ser colocados na posi c ao indicada sem-
pre. H a outras solu c oes.
Revista da ORM/SC n
o
3, 2006
Solu c oes 21
Nvel 2
1. Pelas tabelas vemos que corresponde ao nosso 0, pois somado
com qualquer um dos outros algarismos n ao os altera, e que
corresponde ao nosso 1, pois multiplicado com qualquer um dos
outros algarismos n ao os altera.
O primeiro n umero representado por 2 dgitos e (observe que
na tabela da adi c ao este e o primeiro n umero que surge nas linhas
2, 3 e 4). O primeiro n umero com 3 dgitos deve ser ent ao
(corresponderia ao 100em um sistema posicional de 4 algarismo).
Observe que usando a distributividade temos:
x() = ( + )x() = x + x =
+ , pois n ao altera o n umero na multiplica c ao.
Prolongando-se a tabela de adi c ao e f acil ver que
+ =ou seja, x().
Da conclui-se que
x() =
e que
x =, ou seja, e o quadrado de
.
Esse raciocnio (com as propriedades distributiva e associativa) per-
mite ver que as opera c oes nesse sistema podem ser feitas como no
nosso sistema decimal ( como por exemplo: 3 8 = 24, escrevemos o
4 v ao2).

E isso que usaremos, na pr atica, para o pr oximo item.


x =, pois:

x

( + )

Revista da ORM/SC n
o
3, 2006
22 VII ORM (2004)
e + =, pois:

+

2. Seja abcde um n umero de 5 algarismos (a ,= 0). Ent ao a m aquina faz:
abcde bcde edcb edcbx, onde
x = e +d +c +b
Temos ent ao duas possibilidades:
e ,= 0. Neste caso, como x = 2 (para o n umero n ao se alterar), ent ao
e = e + d + c + b, ou seja, d + c + b = 0, que e impossvel (isto se
b ,= 0).
e = 0. Neste caso temos: abcd0 bcd0 dcb dcbx, onde x
deve ser um n umero de 2 algarismos terminado em zero.Tem-se que
x = d + c + b. (Note que aqui b ,= 0 pois, caso contr ario, teramos
x = d +c e x com 3 algarismos). H a ent ao duas possibilidades:
(a) x = 10 = d +c +b. Mas ent ao d = 1, a = d = 1, b = c.
Segue-se que b +c = 9, o que e impossvel, pois b = c.
(b) x = 20 = d +c +b. Ent ao d = 2, a = d = 2, b = c. Segue-se que
b +c = 18 e da b = c = 9 ( unica resposta).
Portanto o unico n umero que n ao e alterado pela m aquina e 29920.
3. Note que 2004 = 167 2
2
3. (167 e primo)
Assim:
167p +q = 167 12, ou q = 167(12 p)
Como q deve ser primo, ent ao 12 p = 1 , ou seja, p = 11.
Logo p = 11 e q = 167.
4. Analisando-se os 3 grupos de 4 caixas, e sabendo que nenhuma caixa est a
vazia, temos:
Revista da ORM/SC n
o
3, 2006
Solu c oes 23
4 kg 6 kg 18 kg
1
1
1
1
unica possibilidade duas possibilidades ?
As duas possibilidades s ao:
1 + 1 + 1 + 3 = 6
1 + 1 + 2 + 2 = 6
No primeiro caso h a pelo menos 7 caixas com exatamente uma barra.
Olhando para os 4 grupos de 3 caixas teremos a unica possibilidade:
7 kg 7 kg 7 kg 7 kg
1 1 1 1
1 1 1 3 (a caixa de 3 kg deve estar aqui)
5 5 5 3 18 kg
No segundo caso temos pelo menos 6 caixas com exatamente uma barra.
Ent ao temos duas possveis solu c oes, olhando nos grupos de 3 caixas:
1 1 1 2 (as caixas com 2 kg devem estar aqui)
1 1 1 2
5 5 5 3
ou
1 1 1 1
1 1 2 2 (as caixas com 2 kg devem estar aqui)
5 5 4 4
Portanto a maior quantidade de ouro em uma caixa e 5 kg, e h a 3 possveis
distribui c oes.
Revista da ORM/SC n
o
3, 2006
24 VII ORM (2004)
5. De 3 de agosto de 1845 a 2 de agosto de 1849 (inclusive), temos 4 anos
completos (sendo 1 bissexto: 1848). Assim temos:
4 365 + 1 = 1461 dias, neste perodo
O Sr. Silva n ao trabalhou todos estes dias mas ele viajou por 3 dias,
da manh a de 18 de julho de 1849 a manh a 21 de julho de 1849, quando
chegou ` a Terra do Sonho. Ele permanece (pelo menos) 10 dias l a: da
manh a de 21 de julho ate a noite de 30 de julho (ou ate a manh a de 31 de
julho). Se ele voltasse para Shangril a no dia 30 de julho (de manh a ou a
noite), ele chegaria l a no dia 2 de agosto (de manh a ou a noite). Assim,
depois que chega a Shangril a saindo de Eldorado (no dia 3 de Agosto de
1845) ate o dia 2 de agosto de 1849 (possvel volta da Terra do Sonho)
passaram-se 1461 dias. Mas
1461 = 27 54 + 3
Assim, o navio j a haver a passado 3 dias antes, ou seja, no dia 30 de
julho. O pr oximo navio passar a 27 dias depois, ou seja, no dia 26 de
agosto. Assim, Sr. Silva deve partir de Shangril a no dia 23 de agosto
(pela manh a), para chegar no dia 26 de agosto, pela manh a, e tomar o
navio neste mesmo dia no nal da tarde.
Voce Sabia? Os n umeros transcendentes s ao os n umeros que n ao
s ao algebricos. N ao existe nenhum polin omio de coecientes inteiros
de que sejam raiz. O n umero , por exemplo, e um n umero
transcendente porque n ao se pode obte-lo como raiz de nenhum
polin omio de coecientes inteiros. Os n umeros transcendentes s ao
innitos e h a muito mais n umeros transcendentes do que n umeros
algebricos (que s ao aqueles que se podem obter como raiz de um
polin omio de coecientes inteiros). Raiz de 3 e um n umero
algebrico, j a que e solu c ao da equa c ao x
2
3 = 0.
Revista da ORM/SC n
o
3, 2006
Solu c oes 25
Nvel 3
1.
C
D E
A
B
Tracemos BD. Como BE e altura e
mediana do ABD, ent ao AB = BD.
Mas AB = 2 = AD. Logo, o ABD e
equil atero. Assim, E

BD = 30
o
. Segue
que D

BC = 30
o
(pois E

BC = 60
o
) e,
como D

CB = 60
o
, o BCD e ret angulo
em D e, portanto, semelhante ao ABE.
Mas BE
2
= AB
2
AC
2
= 4 1 = 3. Logo,
BE =

3.
Assim, da semelhan ca, temos:
BD
BE
=
CD
AE

3
=
CD
1
CD =
2

3
3
.
Assim:
A
ABD
=
2

3
2
=

3
e
A
BCD
=
2
2

3
3
2
=
2

3
3
.
Portanto, A
ABCD
=

3 +
2

3
3
=
5

3
3
.
2. (i) Pelas tabelas, vemos que corresponde ao nosso 0, pois ele so-
mado a qualquer um dos outros algarismos n ao os altera, e que
corresponde ao nosso 1, pois multiplicado por qualquer um dos
outros algarismos ele n ao os altera.
(ii) O primeiro n umero representado por dois dgitos e (observe
que, na tabela da adi c ao, este e o primeiro n umero que surge nas
linhas 2, 3 e 4). O primeiro n umero com 3 dgitos deve ser, ent ao,
Revista da ORM/SC n
o
3, 2006
26 VII ORM (2004)
(corresponderia a 100 em um sistema posicional de 4 al-
garismos).
(iii) Observe que usando a distributividade temos:
() = (+ ) () = + =
+ ,, pois n ao altera o n umero na multiplica c ao.
Prolongando-se a tabela da adi c ao, e f acil ver que
+ =, ou seja, () =.
Da, conclui-se que () = e que () () =,
ou seja, e o quadrado de . Esse raciocnio (com as pro-
priedades distributiva e associativa) permite ver que as opera c oes
nesse sistema podem ser feitas como no nosso sistema decimal (como,
por exemplo, 3 8 = 24: escrevemos o 4 e v ao 2).

E isso que
usaremos no pr oximo item.
(iv) () + () + () = , pois:
() ()

+


(v) ( ) = () , pois:



( + )


Ent ao, = .
3. Seja abcdef um n umero de seis algarismos (com a ,= 0). Vejamos:
abcdef bcdef fedcb fedcbx.
Revista da ORM/SC n
o
3, 2006
Solu c oes 27
(a) f ,= 0. Ent ao:
(i) ou b ,= 0, mas, neste caso, x = f e x = f + e + d + c + b.
Impossvel.
(ii) ou b = 0. Temos, ent ao:
c ,= 0. Neste caso, a0cdef cdef fedc fedcx, em
que x tem dois algarismos, x = f + e + d + c = 10e + f.
Da, d + c = 9e. Mas ent ao, a = f, b = e = 0 e d + c = 0,
o que faria c = 0. Contradi c ao.
c = 0. Neste caso, a00def def fed fedx, em que
x = f +e +d e tem tres algarismos. Impossvel.
Ent ao, s o resta o caso:
(b) f = 0.
Se b = 0, teremos: a0cde0 cde0 edc edcx, em que x =
e + d + c e tem tres algarismos. Impossvel. Ent ao, se f = 0,
devemos ter b ,= 0. Da: abcde0 bcde0 edcb edcbx, em
que x deve ter dois algarismos e x = e + d + c + b = 10e. Ent ao
d + c + b = 9e. Alem disso, a = e e b = d. A soma m axima de tres
algarismos e 27. Assim, temos:
(i) e = 0; neste caso, d +c +b = 0 d = c = b = 0. Impossvel.
(ii) e = 1; neste caso, a = e = 1 e d +b +c = 9.

_
c = 1 d = b = 4 141410
c = 3 d = b = 3 133310
c = 5 d = b = 2 125210
c = 7 d = b = 1 117110
c = 9 d = b = 0 Impossvel
(iii) e = 2; neste caso, a = e = 2 e d +c +b = 18.

_
c = 0 d = b = 9 290920
c = 2 d = b = 8 282820
c = 4 d = b = 7 274720
c = 6 d = b = 6 266620
c = 8 d = b = 5 258520
(iv) e = 3; neste caso, a=e=3 e d +c +b = 27 b = c = d = 9
399930.
Revista da ORM/SC n
o
3, 2006
28 VII ORM (2004)
4. Para valores de x pr oximos de 1 a primeira fra c ao, em valor absoluto, e
muito grande. O mesmo vale para valores de x pr oximos a 2. Vamos
analisar os valores f(x) para os quatro pontos da lista L mais pr oximos
de 1 e de 2. Os pontos de L s ao: 0,
3
500
,
6
500
, etc. Devemos, ent ao,
tomar o maior valor da lista menor do que 1, o menor valor maior do que
1, o maior valor menor do que 2 e o menor valor maior do que 2. S ao
eles: x
1
=
498
500
, x
2
=
501
500
, x
3
=
999
500
, x
4
=
1002
500
respectivamente.
O valor x
1
est a descartado, pois f(x
1
) < 0. Descartamos tambem x
3
,
pois: [x
3
2[ < [x
3
1[, o que nos d a
1
[x
3
1[
<
1
[x
3
2[
ou
16
[x
3
1[
<
16
[x
3
2[
<
32
[x
3
2[
.
Como
32
x
3
2
< 0, tem-se f(x
3
) < 0.
Vamos analisar f(x
2
) e f(x
4
):
f(x
2
) =
16
501
500
1
+
32
501
500
2
= 16 500
32 500
499
= 16 500
497
499
.
f(x
4
) =
16
1002
500
1
+
32
1002
500
2
=
16 500
502
+
32 500
2
= 16 500
503
502
.
Mas
497
499
< 1 <
503
502
. Logo, f(x
4
) e o maior valor.
5. De 400 q 600, temos 600 q 400.
Portanto 2004 600 2004 q 2004 400 1404 25p 1604
56, 16 p 64, 16.
Os primos nessa faixa s ao 59 e 61. Se p = 59, q = 2004 25 61 = 529.
Porem, 529 = 23
2
. Se p = 61, q = 2004 25 61 = 479. Este n umero e
primo (basta testar a sua divisibilidade por primos ate o 19). Portanto,
p = 61 e q = 479.
Revista da ORM/SC n
o
3, 2006
Premiados 29
Premiados
(em ordem alfabetica por nvel e por tipo de medalha)
Nvel 1
Ouro
Carolina de Paula Peters (Colegio Salesiano de Itaja)
Igor Hinnig Wolniewics (Centro Educacional Menino Jesus)
Marina Freitas Klein (Sociedade Divina Providencia Colegio Sagrada
Famlia)
Natan Cardozo Leal (E.E.B. Orestes Guimar aes)
Vitor Costa Fabris (Associa c ao Beneditina da Providencia - Colegio S ao
Bento)
Prata
Andre Mateus Netto Spillere (Associa c ao Beneditina da Providencia -
Colegio S ao Bento)
Ingrid Knochenhauer (Educand ario Imaculada Concei c ao)
Julia Pinheiro Machado (Stio Escola Sarapiqu a)
Larissa Miranda Hinisch (Colegio Cora c ao de Jesus)
Natsue Eccel Mizubuti (Colegio Santo Ant onio )
Renan Henrique Finder (Colegio dos Santos Anjos)
Bronze
Aline Peterle (E.E.B. Municipal Aurora Peterle)
Camille Fiamoncini Mattos (Educand ario Imaculada Concei c ao)
Eduardo Machado Capaverde (Colegio Cora c ao de Jesus)
Revista da ORM/SC n
o
3, 2006
30 VII ORM (2004)
Guido Quint Tonelli Santos (Educand ario Imaculada Concei c ao)
Jenifer Wegert (IMA - Instituto Maria Auxiliadora)
Men cao Honrosa
Alexandre Schmidt Ferreira (Colegio Murialdo)
Ana Carolina Paterno (Escola Municipal Governador Pedro Ivo Campos)
Ana Luiza de Amorim (Sociedade Divina Providencia Colegio Sagrada
Famlia)
Ana Paula de Assis Schimidt (Centro Educacional Menino Jesus)
Beatriz Luzia Wetzel (E. E. F. Professor Emir Ropelato)
Bernardo de Sousa Valverde (Educand ario Imaculada Concei c ao)
Betina Leit ao Mehl (Colegio Nova Era)
Brenda Schmitt de Araujo de Mattos (Colegio de Aplica c ao da UFSC)
Bruno de Almeida L. C. Silva (Colegio Catarinense)
Bruno de Brida (Alpha Objetivo)
Carlos Eduardo Rosar K os Lassance (Colegio Catarinense)
Carolina Brandt (Colegio Supera c ao)
Carolina de Borba Albino (IMA - Instituto Maria Auxiliadora)
Denise Albertazzi Gon calves (Centro Educacional Menino Jesus)
Douglas Paute (Escola Municipal Governador Pedro Ivo Campos)
Eduardo Biscoli Brand ao (Colegio Supera c ao)
Eduardo Luis Festa (

Aster Centro Educacional)


Eduardo Santos da Silveira (Associa c ao Beneditina da Providencia - Colegio
S ao Bento)
Revista da ORM/SC n
o
3, 2006
Premiados 31
Elton Kasmirski (Colegio Nova Era)
Eun Sol Cho (Colegio Nossa Senhora de Fatima)
Felix Li Han Huang (

Aster Centro Educacional)


Gabriel Fischer de Moraes (Colegio Santo Ant onio )
Gabriel Thom (Colegio Sinodal Ruy Barbosa)
Gabriela Ribeiro Sumar (Colegio Cora c ao de Jesus)
Gilson Vilain Machado (Colegio Dom Jaime C amara)
Gustavo David Ludwig (Colegio Cora c ao de Jesus)
Gustavo Della Bruna N. (Colegio Nossa Senhora de Fatima)
Hermano Roepke (E. E. F. Professor Emir Ropelato)
Igor Piacentini Coelho da Costa (Colegio de Aplica c ao da UFSC)
Isabella Sandrini Pizzolatti (Conjunto Educacional Dr. Blumenau)
Iveraldo Carlos Machado Junior (SOCIESC - Soc. Educ. de Santa Cata-
rina)
Jaqueline Witt Garzillo (Colegio Cenecista Jose Elias Moreira)
Jenifer Milbratz (E. E. F. Professor Emir Ropelato)
Joana dos Santos Copetti (Colegio Murialdo)
Jo ao Marcos N. Coelho (Colegio Murialdo)
Juliane Bonetti (Colegio Elisa Andreoli)
Larissa Borges Marthendal (Colegio Cora c ao de Jesus)
Leonardo Flores Zambaldi (Colegio Tradi c ao)
Leticia Perini (E. E. F. Professor Emir Ropelato)
Luis Gustavo Longen (Colegio Sinodal Ruy Barbosa)
Revista da ORM/SC n
o
3, 2006
32 VII ORM (2004)
Matheus de Bona dos Santos (Centro Educacional Roda Pi ao Ltda )
Paula Azevedo do Nascimento (Colegio da Lagoa)
Pedro Luis Rissoli (Colegio dos Santos Anjos)
Rafael Rogerio Santos (Centro Educacional Roda Pi ao Ltda )
Renato Klueger Junior (Colegio Nossa Senhora de Fatima)
Rodrigo Gamba (Colegio Supera c ao)
Samia Pauli Fiates (Colegio Cora c ao de Jesus)
Sergio Feldemann de Quadros (Colegio Salesiano de Itaja)
Tainara Helena de Amorim (Sociedade Divina Providencia Colegio Sagrada
Famlia)
Thaminne Silveira (Colegio Cenecista Jose Elias Moreira)
Unrio Machado dos Santos J unior (Colegio Elisa Andreoli)
Vanessa Martins Rosa (Colegio Cora c ao de Jesus)
Victor Abouhatem (Colegio da Lagoa)
Vinicius da Costa Mohr (Colegio Cenecista Jose Elias Moreira)
Vincius Rios Fuck (Colegio Elisa Andreoli)
Yuri da Silva Villas Boas (Colegio Catarinense)
Nvel 2
Ouro
Danilo Nunes do Carmo (Colegio de Aplica c ao da UFSC)
Jessica Cardoso dos Santos (Alpha Objetivo)
Leonardo Pinheiro Samar ao (Colegio de Aplica c ao da UNIVALI)
Petrius Paulo Tambosi (Conjunto Educacional Dr. Blumenau)
Revista da ORM/SC n
o
3, 2006
Premiados 33
Ruan Ricardo Rengel (Conjunto Educacional Dr. Blumenau)
Tiago Madeira (Colegio Salesiano de Itaja)
Prata
Caio Andrezzo (E.M.E.F. Albano Kanzler)
Cristine Ribas (Alpha Objetivo)
Luiz Fernando de Amorim Joll Embeck (Colegio de Aplica c ao da UFSC)
Tatiana Cristine de Amorim (Sociedade Divina Providencia Colegio Sagrada
Famlia)
Vanessa Fischer dos Santos (Colegio Santo Ant onio )
Victor Rodolfo Pereira Lopes (Colegio Catarinense)
Bronze
Ana Paula Peixoto Bittencourt (Colegio de Aplica c ao da UNIVALI)
Douglas Maiola (Escola Municipal Governador Pedro Ivo Campos)
Giuliana Sardi Venter (Escola Bar ao do Rio Branco)
Lucas Bet da Rosa Orssatto (Colegio de Aplica c ao da UFSC)
Paula do Vale Pereira (Educand ario Imaculada Concei c ao)
Pietro Jose Bertuzzi (Colegio Elisa Andreoli)
Vinicius da Silveira Segali (Colegio de Aplica c ao da UFSC)
Men cao Honrosa
Adriano Reinaldo Timm (Escola Municipal Padre Martinho Stein)
Allan Kipper Marquetti (Colegio Cora c ao de Jesus)
Amanda Pereira Medeiros (Colegio Murialdo)
Ana Luiza Pagani Fonseca (Centro Educacional Menino Jesus)
Revista da ORM/SC n
o
3, 2006
34 VII ORM (2004)
Antonio Cezar Quevedo Goulart Filho (Colegio Catarinense)
Bruna Iha (Colegio da Lagoa)
Bruno Santos Vieira (Colegio Dehon)
Camila Costa Hermani (Colegio Rogacionista Pio XII)
Camila Tormena (Colegio Santo Ant onio)
Christian Juliano Pereira (Colegio Cenecista Jose Elias Moreira)
Cristine Saibert (Centro Educacional Roda Pi ao Ltda)
Douglas Machado Vieira (Colegio Catarinense)
Erica Schmitt Mafra (Colegio de Aplica c ao da UNIVALI)
Felipe Alves de Souza (Colegio Elisa Andreoli)
Fernanda Pessoa de Carvalho (Centro Educacional Roda Pi ao Ltda )
Gabriel Nunes (Colegio Supera c ao)
Gabrielle C. Rocha (Escola Bar ao do Rio Branco)
Geisla Thamara de Abreu (Colegio :Dom Bosco)
Guilherme Kawase Falk (Educand ario Imaculada Concei c ao)
Helosa Helena Rodrigues (Colegio Dom Jaime C amara)
Jeerson da Silva Decom (Colegio Santo Ant onio)
Jessica Pauli de Castro Bonson (Educand ario Imaculada Concei c ao)
Julie Sabine Holetz Brandes (Escola Municipal Erwin Prade)
Leonardo Bruno Pereira de Moraes (Stio Escola Sarapiqu a)
Leonardo Sgnaolin (Colegio Dom Jaime C amara)
Libin Yang (Colegio Catarinense)
Lourival Tenfen Junior (Colegio Cenecista Jose Elias Moreira)
Revista da ORM/SC n
o
3, 2006
Premiados 35
Luany Tamires Fiedler (E.M.E.F. Maria Nilda Salai St ahelin)
Lucas Boppre Niehues (SOCIESC - Soc. Educ. de Santa Catarina)
Lucas Werner (Colegio DEHON/UNISUL)
Maicow Willian Zanon (E. E. F. Professor Emir Ropelato)
Marcos Aurelio S. Timmermann (Colegio Nova Era)
Marcos Eduardo de Farias (Centro Educacional Roda Pi ao Ltda)
Maria Tereza Amorim Falc ao (Colegio Santo Ant onio)
Mayra Trierveiler (Conjunto Educacional Dr. Blumenau)
Paulo Henrique Cardozo (Centro Educacional Menino Jesus)
Philippi Farias Rachadel (Educand ario Imaculada Concei c ao)
Renan Machado Capaverde (Colegio Cora c ao de Jesus)
Renata da Silva Heying (Escola Municipal Erwin Prade)
Ricardo Maurino Melo (Centro Educacional Roda Pi ao Ltda)
Ricardo Soares Schwingel (Colegio de Aplica c ao da UNIVALI)
Theodor Wilhelm Adler (Colegio da Lagoa)
Vitor Luis Pereira (Colegio dos Santos Anjos)
Vitos Cesar Kanitz (Escola Municipal Erwin Prade)
Wagner Daufenbach do Amaral (Associa c ao Beneditina da Providencia -
Colegio S ao Bento)
Yone Ecal Mizubuti (Colegio Santo Ant onio)
Revista da ORM/SC n
o
3, 2006
36 VII ORM (2004)
Nvel 3
Ouro
Alex Carlos Schmidt (SOCIESC - Soc. Educ. de Santa Catarina)
Bruno Pereira Dias (Colegio Catarinense)
Felipe Paupitz Schlichting (Colegio Cora c ao de Jesus)
Guilherme Rohden Echelmeier (Colegio de Aplica c ao da UNIVALI)
Gustavo Henrique Nihei (Colegio Dom Jaime C amara)
Robert da Silva Bressan (Colegio Dehon)
Prata
Bruno Leonardo Schneider (Colegio Dom Jaime C amara)
Henrique Antonio Calbo Perdoncini (SOCIESC - Soc. Educ. de Santa
Catarina)
Luckas Frigo Furtado (Colegio Vis ao)
Rafael Peralta Muniz Moreira (Colegio Catarinense)
Vincius Bastos Farias (Colegio Dom Jaime C amara)
Bronze
Alan Schmitt (Colegio de Aplica c ao da UNIVALI)
Cindy Dalfovo (SOCIESC - Soc. Educ. de Santa Catarina)
Fabrcio Marques Correa (Associa c ao Beneditina da Providencia - Colegio
S ao Bento)
Jose Artur Silveira Teixeira (Colegio Dom Jaime C amara)
Karine Piacentini Coelho da Costa (Centro Federal de Educa c ao Tec-
nol ogica de Santa Catarina )
Revista da ORM/SC n
o
3, 2006
Premiados 37
Kellen Trilha Schappo (Curso e Colegio Energia)
Marcelo Adriano de Macedo (SOCIESC - Soc. Educ. de Santa Catarina)
Paulo Henrique Baumann (Colegio Catarinense)
Pedro Henrique Boscardin de Araujo (Curso e Colegio Energia)
Willian Alexandre Suguino (Curso e Colegio Energia)
Men cao Honrosa
Ana Paula Alves Monteiro (Colegio Dom Jaime C amara)
Andre Krummenauer (Colegio Dom Jaime C amara)
Andre Luiz Quintino (SOCIESC - Soc. Educ. de Santa Catarina)
Andre Luiz Thieme (Colegio de Aplica c ao da UNIVALI)
Andre Luiz Tomelin (Colegio Catarinense)
Andrey Jose Taner Fraga (Colegio Henry Ford Ltda)
Bruno Bernardo Teixeira (Colegio Carrossel)
Carla Medina Ribeiro Protta (Centro Federal de Educa c ao Tecnol ogica
de Santa Catarina)
Carolina Barbi Linhares (Colegio Salesiano de Itaja)
Cintya Kazue Sakamoto (Centro Federal de Educa c ao Tecnol ogica de
Santa Catarina)
Diego Bonkowski de La Sierra Audired (SOCIESC - Soc. Educ. de
Santa Catarina)
Diogo Alexandre Parente (SOCIESC - Soc. Educ. de Santa Catarina)
Elisa Rego Mendes (Alpha Objetivo)
Felipe Borges Alves (Curso e Colegio Energia)
Felipe Francisco (Alpha Objetivo)
Revista da ORM/SC n
o
3, 2006
38 VII ORM (2004)
Felippe Frigo Furtado (Colegio Vis ao)
Gabriele Tsch a (E.E.B. Professor Tulio Scheimantel)
Huilton Estevo Martins (Colegio Elisa Andreoli)
Israel Pereira (Alpha Objetivo)
Jaqueline Homann (Colegio Henry Ford Ltda)
Jer onimo Moraes Gomes (Centro Federal de Educa c ao Tecnol ogica de
Santa Catarina )
Jo ao Frederico Martendal Neto (Colegio Elisa Andreoli)
Jose Roberto Cordeiro (Colegio Catarinense)
Kananda Silvano Silveira (Colegio de Aplica c ao da UFSC)
Laura Luisa Medeiros de Souza (Kumon - Joa caba)
Leonardo Silva Alves (Colegio Dom Jaime C amara)
Luiza Magalh aes de Oliveira (Colegio Catarinense)
Luiza Mamigonian Bessa (Centro Federal de Educa c ao Tecnol ogica de
Santa Catarina)
Nelisa Helena Rocha (Curso e Colegio Energia)
Olivan Bittencourt de Carvalho (SOCIESC - Soc. Educ. de Santa Cata-
rina)
Pedro Neves Schondermark (Colegio Elisa Andreoli)
Priscila Teixeira Quaini (Colegio Catarinense)
Ramon Rodrigues Rita (Colegio Catarinense)
Ricardo M uller (SOCIESC - Soc. Educ. de Santa Catarina)
Roberta Muriel Longo Roepke (Colegio Henry Ford Ltda)
Rodrigo de Rodrigues (Alpha Objetivo)
Revista da ORM/SC n
o
3, 2006
Premiados 39
Roger Savoldi Roman (Colegio Energia - Itaja)
Sergio Brillinger Novello (Colegio Catarinense)
Thiago Rossi Trojan (SOCIESC - Soc. Educ. de Santa Catarina)
Thomas Eduardt Hafemann (Conjunto Educacional Dr. Blumenau)
Yuri Kaszubowski Lopes (SOCIESC - Soc. Educ. de Santa Catarina)
Revista da ORM/SC n
o
3, 2006
40 VII ORM (2004)
Escolas Participantes
1. Alpha Objetivo (S ao Jose)
2. Associa c ao Beneditina da Providencia - Colegio S ao Bento (Crici uma)
3. Associa c ao Francisquense de Ensino (S ao Francisco do Sul)
4.

Aster Centro Educacional (Balne ario Camburi u)
5. CEFETSC-UNED/SJ (S ao Jose)
6. Cefrai Centro Educacional Fraiburgo (Fraiburgo)
7. Centro Educacional Bom Jesus (Palho ca)
8. Centro Educacional Extens ao Ltda (Sombrio)
9. Centro Educacional Integrado Jaraguaense - CEIJ (Jaragu a do Sul)
10. Centro Educacional Menino Jesus (Florian opolis)
11. Centro Educacional Prncipe Ali (S ao Jose)
12. Centro Educacional Roda Pi ao Ltda (Palho ca)
13. Centro Educacional Tmb o S/A CETISA (Timb o)
14. Centro Educaional Integral Mundo da Magia (Tijucas)
15. Centro Federal de Educa c ao Tecnol ogica de Santa Catarina (Florian opolis)
16. Colegio Atl antico (Itapema)
17. Colegio Atl antico Sul (Balne ario Camburi u)
18. Colegio Bom Jesus Santo Ant onio (Blumenau)
19. Colegio Carrossel (Palho ca)
20. Colegio Catarinense (Florian opolis)
21. Colegio Cenecista Jose Elias Moreira (Joinville)
22. Colegio Cora c ao de Jesus (Florian opolis)
Revista da ORM/SC n
o
3, 2006
Escolas Participantes 41
23. Colegio Criativo (Florian opolis)
24. Colegio Cruz e Sousa (Florian opolis)
25. Colegio da Lagoa (Florian opolis)
26. Colegio da Univille (Joinville)
27. Colegio de Aplica c ao da UFSC (Florian opolis)
28. Colegio de Aplica c ao da UNIVALI (Itaja)
29. Colegio de Aplica c ao UNIVALI - Tijucas (Tijucas)
30. Colegio Decis ao (Florian opolis)
31. Colegio Dehon (Tubar ao)
32. Colegio Dehon/UNISUL (Ararangu a)
33. Colegio Dom Bosco (Rio do Sul)
34. Colegio Dom Jaime C amara (S ao Jose)
35. Colegio dos Santos Anjos (Joinville)
36. Colegio Elisa Andreoli (S ao Jose)
37. Colegio Energia - Brusque (Brusque)
38. Colegio Energia - Itaja (Itaja)
39. Colegio Espa co (Bra co do Norte)
40. Colegio Evidencia (Balne ario Camburi u)
41. Colegio Henry Ford Ltda (Timb o)
42. Colegio Marista (Crici uma)
43. Colegio Marista Frei Rogerio (Joa caba)
44. Colegio Murialdo (Ararangu a)
45. Colegio Nossa Senhora de F atima (Florian opolis)
Revista da ORM/SC n
o
3, 2006
42 VII ORM (2004)
46. Colegio Nova Era (Joinville)
47. Colegio Rogacionista Pio XII (Crici uma)
48. Colegio Salesiano Itaja (Itaja)
49. Colegio Santo Antonio (Joinville)
50. Colegio Sigma (Lages)
51. Colegio Sinodal Ruy Barbosa (Rio do Sul)
52. Colegio Supera c ao (Videira)
53. Colegio Tradi c ao (Florian opolis)
54. Colegio Unicado (Gaspar)
55. Colegio Vis ao (S ao Jose)
56. Conjunto Educacional Dr Blumenau (Pomerode)
57. Curso E Colegio Dom Bosco (S ao Jose)
58. Curso e Colegio Energia (Crici uma)
59. Curso e Colegio Energia (Tubar ao)
60. Curso e Colegio Energia (Ararangu a)
61. Curso e Colegio Energia (Florian opolis)
62. Curso e Colegio Energia - Joinville (Joinville)
63. Curso e Colegio Lavoisier (S ao Jose)
64. E.E.B. Humberto Hermes Homann (Nova Veneza)
65. E.E.M. Professor Roberto Grand (S ao Bento do Sul)
66. E.E.B. Dolores Luzia S. Krauss (Gaspar)
67. E.E.B. Ceclia Rosa Lopes (S ao Jose)
68. E.E.B. Conselheiro Manoel Philippi (

Aguas Mornas)
Revista da ORM/SC n
o
3, 2006
Escolas Participantes 43
69. E.E.B. Dr. Paulo Medeiros (Joinville)
70. E.E.B. Jo ao Gaya (Luiz Alves)
71. E.E.B. Nereu Ramos (Itaja)
72. E.E.B. Professor J ulio Scheidemantel (Timb o)
73. E.E.B. Prof
o
Jo ao Martins Veras (Joinville)
74. E.E.B. Prot asio Joaquim da Cunha (Sombrio)
75. E.E.B. Tenente Anselmo Jose Hess (Luiz Alves)
76. E.E.B.Professor Jose Rodrigues Lopes (Garopaba)
77. E.E.M. Dr. Ruben Roberto Schmidlin (Joinville)
78. E.M.E.F Rodolpho Dornbusch (Jaragu a do Sul)
79. E.M.E.F. Maria Nilda Salai St ahelin (Jaragu a do Sul)
80. Educand ario Imaculada Concei c ao (Florian opolis)
81. E.E.B. Coronel Ant onio Lehmkuhl (

Aguas Mornas)
82. E.E.B. General Osvaldo Pinto da Veiga (Capivari de Baixo)
83. E.E.B. Get ulio Vargas (Florian opolis)
84. E.E.B. Osvaldo Aranha (Joinville)
85. E.E.B. Professor Henrique Stodieck (Florian opolis)
86. E.E.B. Ruy Barbosa (Timb o)
87. E.E.B. Professor Heriberto Joseph M uller (Blumenau)
88. E.E.F. Dom Jaime de Barros C amara (Palho ca)
89. Escola de Educa c ao B asica Raulino Horn (Indaial)
90. Escola Agrotecnica Federal de Rio do Sul (Rio do Sul)
91. Escola Agrotecnica Federal de Sombrio (Santa Rosa do Sul)
Revista da ORM/SC n
o
3, 2006
44 VII ORM (2004)
92. Escola Bar ao do Rio Branco (Blumenau)
93. Escola B asica Municipal Beatriz de Souza Brito (Florian opolis)
94. Escola B asica Municipal Paulo Rizzieri (I cara)
95. Escola Basica Municipal Tranquillo Pissetti (I cara)
96. Escola B asica Professor Leopoldo Hanof (Orleans)
97. Escola de Educa c ao B asica de Lages (Lages)
98. Escola de Educa c ao B asica Francisco Eberhardt (Joinville)
99. Escola de Educa c ao B asica Frei Policarpo (Gaspar)
100. Escola de Educa c ao B asica Henrique Estefano Koerich (Palho ca)
101. Escola de Educa c ao B asica Irm a Maria Teresa (Palho ca)
102. Escola de Educa c ao B asica Jo ao Frassetto (Crici uma)
103. Escola De Educa c ao B asica Municipal Aurora Peterle (Sider opolis)
104. Escola de Educa c ao B asica Jo ao Colin (Joinville)
105. Escola de Educa c ao B asica Orestes Guimar aes (S ao Bento do Sul)
106. Escola de Educa c ao B asica Prefeito Avelino Muller (Bigua cu)
107. Escola de Educa c ao B asica S ao Jo ao (Agrol andia)
108. Escola de Ensino B asico Presidente Juscelino Kubitschek (S ao Jose)
109. Escola de Ensino Fundamental Professor Emir Ropelato (Timb o)
110. Escola de Ensino Medio Alberto Bauer (Jaragu a do Sul)
111. Escola Din amica (Florian opolis)
112. Escola Municipal de Ensino Fundamental Albano Kanzler (Jaragu a do
Sul)
113. Escola Municipal de Ensino Fundamental Max Schubert (Jaragu a do Sul)
Revista da ORM/SC n
o
3, 2006
Escolas Participantes 45
114. Escola Municipal E.F. Itinerante Maria Alice Wolf Souza (Lages)
115. Escola Municipal Erwin Prade (Timb o)
116. Escola Municipal Governador Ivo Silveira (Balne ario Camburi u)
117. Escola Municipal Governador Pedro Ivo Campos (Joinville)
118. Escola Municipal Maurcio Germer (Timb o)
119. Escola Municipal Padre Martinho Stein (Timb o)
120. Escola Municipal Professora Karin Barkemeyer (Joinville)
121. IMA - Instituto Maria Auxiliadora (Riodo Sul)
122. KUMON - Curso (Florian opolis)
123. KUMON - Joa caba (Joa caba)
124. KUMON - Unidade Porto Uni ao (Porto Uni ao)
125. Kumon - Unidade S ao Bento do Sul (S ao Bento do Sul)
126. Sistema de Ensino Lideran ca (Itaja)
127. Stio Escola Sarapiqu a (Florian opolis)
128. Sociedade Divina Providencia Colegio Sagrada Famlia (Blumenau)
129. Sociedade Educacional de Santa Catarina (Joinville)
Revista da ORM/SC n
o
3, 2006
46 VII ORM (2004)
Revista da ORM/SC n
o
3, 2006
Artigo
48 Artigo
Revista da ORM/SC n
o
3, 2006
Os N umeros e o Innito 49
Os N umeros e o Innito
Ivan Pontual Costa e Silva
Dep. de Matem atica, Universidade Federal de Santa Catarina
CEP: 88.040-900, Florian opolis-SC
Um Pouco de Hist oria
A ideia b asica de n umero e pelo menos t ao antiga quanto o homo sapiens,
e provavelmente j a era conhecida por alguns de seus ancestrais homindeos.

E
tambem uma das primeiras ideias matem aticas que temos, ainda crian cas. Nos
dias de hoje, mesmo em sociedades onde essa concep c ao n ao atingiu qualquer
grau de sostica c ao (como por exemplo em certos grupos indgenas ou em
certas tribos africanas) podemos perceber a existencia das no c oes b asicas de
um, dois, muitos. No entanto, nos defrontamos com diculdades consider aveis
se tentamos denir n umero mais precisamente. Se nos perguntamos o que
e n umero?, freq uentemente sentimos que justamente por ser uma no c ao t ao
b asica e familiar se torna difcil dar uma resposta adequada.
A no c ao de innito certamente e mais sutil, mas ainda bastante familiar.
Assim como no caso dos n umeros, e difcil caracteriz a-la adequadamente em
palavras. Em Filosoa, faz-se uma distin c ao entre innito potencial, que cor-
responde a um processo que continua sem cessar, e innito atual, que e um
innito est atico, pleno e acabado. Um exemplo do primeiro tipo de innito
vem se, come cando do n umero 1, passamos a somar mais um. Esse pro-
cesso, ` a parte das obvias limita c oes fsicas, em princpio n ao terminaria jamais,
isto e, nunca chegamos a um n umero que seja o maior. Mas ainda assim,
em geral n ao pensamos no innito como um n umero; o innito nesse exemplo
e, portanto, potencial. Essas concep c oes de innito devem-se principalmente
a Arist oteles(384-322 a.C.), que via v arios exemplos de innito potencial na
Natureza, como o ciclo (assim ele julgava) das esta c oes do ano. Arist oteles,
contudo, negava que pudesse existir innito atual. Uma reta em geometria
costumava ser vista como outro exemplo de innito potencial: n ao importa
Revista da ORM/SC n
o
3, 2006
50 Artigo
o quanto se caminha sobre a reta, pode-se sempre ir alem.
1
Usando a ideia
aristotelica de innito potencial, o matem atico grego Eudoxo de Cnido(408-
355 a.C.) elaborou aquilo que se tornaria a semente do C alculo Integral seculos
depois, o metodo da exaust ao.
2
Arquimedes(287-212 a.C.) usou esse metodo
para calcular areas e/ou volumes de guras como o crculo e a esfera. Fil osofos
como Plotino(205-270 a.C.), por outro lado, defendiam a existencia metafsica
do innito atual, a ser conhecido atraves de insights msticos.
Se o leitor sente que essas no c oes s ao vagas e controversas, n ao est a sozi-
nho. Os matem aticos permaneceram, em sua maioria, longe de tais discuss oes
metafsicas. Ainda assim, com o advento do C alculo Diferencial e Integral no
sec. XVII, eles passaram a trabalhar sistematicamente com conceitos bastante
mal-denidos e relacionados ao innito, como a ideia de innitesimais (quan-
tidades que seriam n ao-nulas, mas menores do que qualquer n umero) e somas
com uma innidade de termos. Em uma polemica que se tornou famosa em
sua epoca, o bispo irlandes G. Berkeley(1685-1753), em sua obra O Analista,
criticou violentamente (e com bastante pertinencia) o uso descuidado dessas
no c oes difusas. Felizmente, os matem aticos continuaram seu trabalho sem se
importar com tais crticas, e realizaram tremendos avan cos. De fato, e uma
caracterstica peculiar da ciencia que o rigor excessivo imposto no incio de uma
investiga c ao sufoca a imagina c ao e a criatividade. No entanto, de um modo
geral julgava-se, em concord ancia com Arist oteles, que s o o innito potencial
teria lugar na Matem atica.
No sec. XIX, porem, tendo a Matem atica alcan cado um enorme desen-
volvimento, sentiu-se a necessidade de deni c oes mais precisas e um cuidado
maior com o rigor das demonstra c oes. Fundamental na epoca foi o movimento
de aritmetiza c ao da An alise, promovida por nomes como B. Bolzano(1781-
1848), A.-L. Cauchy (1789-1857) e K. Weierstrass(1815-1897), para tornar mais
rigorosas as bases do C alculo Diferencial e Integral, libertando-o do conceito
de innitesimal. Esse objetivo foi alcan cado denindo-se adequadamente limite
de fun c oes, continuidade, series innitas, etc. em termos de propriedades dos
n umeros reais. O passo seguinte para estabelecer a An alise Matem atica em
bases s olidas seria uma melhor compreens ao matem atica dos n umeros reais.
1
Ali as, os termos potencial e atual derivam-se das no c oes tecnicas em Filosoa de ato
e potencia. Em particular, como j a deve estar claro do contexto, atual n ao se refere ao
presente momento.
2
Veja, por exemplo, as Refs. [1] para uma descri c ao do metodo da exaust ao, e para mais
detalhes hist oricos do que apresentado aqui.
Revista da ORM/SC n
o
3, 2006
Os N umeros e o Innito 51
Os pioneiros nessa investiga c ao foram principalmente J. Dedekind(1831-
1916) e G. Cantor(1845-1918). Tornou-se possvel construir, em um sentido
tecnico preciso que n ao e possvel discutir aqui, os n umeros reais a partir dos
n umeros racionais, construir estes a partir dos n umeros inteiros, e estes ultimos
a partir dos n umeros naturais
3
. Em cada passo, podem ser denidas, de
um modo preciso, as opera c oes de soma, produto, etc., e demonstradas, como
teoremas, suas propriedades b asicas (o leitor interessado pode consultar, por
exemplo, a Ref. [2]). O desenvolvimento l ogico nal ent ao passou a depender
exclusivamente dos n umeros naturais.
Seria possvel caracterizar os n umeros naturais de forma matematicamente
precisa? A resposta positiva a essa pergunta assumiu sua forma denitiva com
os axiomas de Peano, criados pelo matem atico e l ogico italiano G. Peano(1858-
1932)
4
, a partir dos quais toda a aritmetica dos n umeros naturais pode ser
obtida. Peano usou os termos primitivos n umero (natural), zero e suces-
sor de e cinco axiomas envolvendo estes termos . Veja a Ref.[2] para uma
apresenta c ao e discuss ao dos axiomas de Peano.
Motivado por seus estudos em An alise Matem atica, Cantor criou em sua
epoca uma nova disciplina matem atica, a Teoria dos Conjuntos
5
, que veio a
se tornar a base de grande parte da Matem atica moderna, e revolucionou o
ensino dessa disciplina. No curso de suas descobertas, Cantor (e, em menor
grau, Dedekind) deu a primeira caracteriza c ao precisa da no c ao de innito em
Ciencia, e mostrou que de fato existem tipos diferentes de innito! Falamos hoje
do conjunto N dos n umeros naturais, e ao pensar na totalidade dos n umeros,
estamos de fato introduzindo o innito atual na Matem atica. Embora isso
3
O leitor com pouca experiencia em Matem atica superior pode car espantado ao ouvir
falar em construirn umeros. O que se quer dizer e que se denem certos objetos matem aticos
cujas propriedades mimetizam as propriedades usuais dos n umeros, tornando-os modelos
matem aticos, ou vers oes abstratas, do conceito intuitivo de n umero. S ao uteis por terem a
precis ao que falta ao conceito intuitivo. Esse tipo de processo de abstra c ao est a no cora c ao
da Matem atica.
4
Lembramos ao leitor que axiomas s ao proposi c oes n ao demonstradas, envolvendo apenas
certos termos n ao denidos, os termos primitivos, a partir das quais se podem derivar, atraves
das regras da l ogica, as proposi c oes demonstradas ou teoremas. Esse e o chamado metodo
axiom atico. Toda a Matem atica moderna, bem como partes de algumas outras ciencias,
como a Fsica, baseiam-se no metodo axiom atico.
5
Cantor elaborou a Teoria dos Conjuntos de forma relativamente intuitiva, que e a maneira
como esta disciplina e ensinada nas escolas. Um conjunto de axiomas para a Teoria dos Con-
juntos foi apresentado pelo matem atico alem ao E. Zermelo(1871-1959) entre 1904-08, pos-
teriormente desenvolvidos por T.Sk olem(1887-1963) e A. Fraenkel(1891-1965). Neste artigo,
usaremos uma abordagem intuitiva ` a la Cantor.
Revista da ORM/SC n
o
3, 2006
52 Artigo
pare ca t ao prosaico para n os hoje, que desde pequenos estudamos Matem atica
na escola atraves da no c ao de conjunto, as descobertas de Cantor foram ex-
tremamente revolucion arias para a epoca, e s ao reconhecidas como estando
entre as maiores conquistas intelectuais da hist oria humana.
O restante do Artigo e como segue:
Na Se c ao 2, mostraremos que o conjunto N dos n umeros naturais, o conjunto
Z dos n umeros inteiros e o conjunto Q dos n umeros racionais s ao innitos de
um mesmo tipo (innito enumer avel), mas o conjunto R dos n umeros reais
pertence a uma classe de innito maior (innito n ao-enumer avel).
Na Se c ao 3, discutiremos a caracteriza c ao de Cantor de conjuntos innitos,
bem como a hierarquia de innitos introduzida por ele.
Finalmente, no Apendice, revisamos as deni c oes b asicas de: fun c ao, fun c ao
injetora, sobrejetora e bijetora.
Um, Dois, Tres, Innito
Para entender o grande insight de Cantor, vamos considerar uma pergunta
simples: o que signica contar ?
Quando crian cas, em geral s o sabemos contar coisas em n umero pequeno,
usando os dedos das duas m aos, por exemplo. Nesse caso, associamos cada
objeto a ser contado com um dedo das m aos, de forma que a cada objeto esteja
associado exatamente um dedo, isto e, de modo que n ao haja dois dedos para
um unico objeto, ou dois objetos para um unico dedo. Conseguimos contar
desse modo somente tantos objetos quanto podemos associar assim aos dedos
das m aos. Se houver mais objetos do que dedos, n ao conseguiremos, somente
usando as m aos, contar esses objetos, mas contaremos assim um subconjunto
pr oprio (isto e um subconjunto que n ao e o conjunto todo) do conjunto desses
objetos. Suponha (com o n umero regular de dedos) que tenhamos um conjunto
O com exatamente 10 objetos, digamos 10 ma c as. Nesse caso, estabeleceremos
o que em Matem atica se chama uma bije c ao, ou correspondencia biunvoca
entre o conjunto dos dedos e o conjunto O de ma c as (veja o Apendice para
uma deni c ao mais precisa): cada dedo corresponde a exatamente uma ma c a,
n ao sobram dedos nem ma c as sem seus associados.
Suponha porem que O tenha 15 elementos. Ent ao n ao poderemos, s o us-
ando os dedos das m aos, estabelecer uma bije c ao. Conseguiremos no m aximo
estabelecer uma bije c ao entre o conjunto dos dedos e um subconjunto pr oprio
Revista da ORM/SC n
o
3, 2006
Os N umeros e o Innito 53
O

O com 10 elementos, isto e, 10 ma c as. Se chamamos de D o conjunto


dos 10 dedos das m aos, podemos ent ao estabelecer uma regra que para cada
elemento de D (isto e, cada dedo) associa um unico elemento de O

. Podemos
pensar nessa associa c ao matematicamente como uma fun c ao f : D O que e
injetora, mas n ao sobrejetora (ver Apendice).
6
Podemos, ` a custa de um ligeiro aumento na abstra c ao, tornar essa descri c ao
mais geral. Seja X um conjunto qualquer. Se X e o conjunto vazio, isto
e, sem elementos, podemos dizer que X tem 0 (zero) elementos, ou que o
n umero de elementos de X e zero. Vamos imaginar agora que X e o conjunto
de estrelas da Via-L actea. Esse conjunto, embora enorme, e nito. Vamos
imaginar, com um esfor co adicional de imagina c ao, que desejemos cont a-lo.
Nesse caso, come camos escolhendo uma das estrelas, e associando a essa estrela
o n umero 1. A seguir tomamos outra estrela e associamos o n umero 2, e
assim sucessivamente, de modo a rotular cada estrela com um n umero diferente.
Teremos assim a seq uencia de n umeros naturais 1, 2, . . . , N ate um n umero
N, correspondendo ` a ultima estrela que tomemos. Nesse caso o natural N e
exatamente o que entendemos pelo n umero de objetos de X. Note que nesse
caso temos uma bije c ao entre os conjuntos 1, 2, . . . , N e X.
Essa discuss ao sugere um fato geral interessante. Para comparar o n umero
de elementos de dois conjuntos nitos, n ao e necess ario cont a-los separada-
mente. Em um onibus, por exemplo, para vericar se h a mais assentos ou
passageiros, n ao e necess ario contar os assentos e os passageiros. Basta fazer
cada passageiro sentar em um assento. Se sobram passageiros, h a menos as-
sentos e mais passageiros, e se sobram assentos, ocorre o oposto. O n umeros
de elementos de dois conjuntos e igual se, e somente se, h a uma bije c ao entre
eles. O genio de Cantor foi notar que isso pode ser imediatamente generalizado
para cole c oes innitas, como discutiremos abaixo.
Subindo ainda uma nota na abstra c ao, podemos tornar essas ideias mais
precisas. Para cada n N

= N 0, denotaremos por I
n
o conjunto
I
n
:= k N : 1 k n = 1, 2, . . . , n.
Temos ent ao a seguinte deni c ao fundamental.
6
Nesse ponto e preciso mencionar novamente que o termo conjunto tem um signicado
tecnico em Matem atica diferente do conceito de cole c ao na linguagem corrente. Embora
intuitivamente seja natural consider a-los como tal, as cole c oes de ma c as e dedos n ao s ao con-
juntos em um sentido matem atico. Isso pode ser uma surpresa para o estudante que, lidando
com ideias intuitivas, identica qualquer cole c ao com conjunto. No entanto, desconsideramos
tais sutilezas em prol da clareza de ideias.
Revista da ORM/SC n
o
3, 2006
54 Artigo
Deni cao: Um conjunto X e dito ser nito se, e somente se, for vazio ou se
existir n N

tal que h a uma bije c ao entre I


n
e X. Se X n ao e nito ent ao X
e dito ser innito.

E possvel mostrar (veja, por exemplo, a Ref.[3]) que, dado um conjunto


n ao-vazio X, se existirem n, n

tais que h a uma bije c ao entre X e cada um


dos I
n
, I
n
, ent ao n = n

. Ou seja, se X e nito, ent ao est a associado a ele de


forma unica um n umero natural n, que e chamado o n umero de elementos de X.
H a v arias outras proposi c oes em [3] que garantem que essa deni c ao e boano
sentido de que captura v arias ideias intuitivas a respeito de conjuntos nitos.
Por exemplo, pode-se provar que todo subconjunto de um conjunto nito X
com n elementos e tambem nito e tem no m aximo n elementos. Tambem se
mostra que se temos conjuntos nitos S
1
, S
2
disjuntos, com m e n elementos
respectivamente, ent ao S
1
S
2
e nito e tem m+n elementos.
Com a deni c ao acima, temos uma caracteriza c ao matematicamente precisa
do que signica contar: ao menos no caso de conjuntos nitos, contar e
estabelecer uma bije c ao com algum subconjunto I
n
de N.
Mas podemos ir alem.

E instrutivo demonstrar, de acordo com essa deni c ao,
o seguinte fato intuitivamente obvio:
Teorema 1 : N n ao e um conjunto nito.
Demonstra cao: Vamos usar redu c ao ao absurdo. Suponha que N fosse nito.
Ent ao existiria n N

tal que h a uma bije c ao entre I


n
e N. No entanto, seja
dada, arbitrariamente, uma fun c ao : I
n
N. Ora, seja p = (1) + +(n).
Claro que p N. Note-se ainda que p > (i) para cada i I
n
. Em particular,
n ao existe i I
n
tal que (i) = p. Mas ent ao n ao e sobrejetora, e portanto
em particular n ao pode ser bijetora. Mas como e arbitr aria, isso mostra que
n ao pode haver uma bije c ao entre I
n
e N, contrariando a hip otese.
Outros exemplos de conjuntos innitos s ao o conjunto Z dos n umeros in-
teiros e o conjunto Q dos n umeros racionais. De fato, como esses conjuntos
contem N, se fossem nitos N tambem teria que ser, contrariando o teorema
que acabamos de demonstrar.
Os conjuntos innitos tem propriedades interessantes e anti-intuitivas. Por
exemplo, o conjunto P dos n umeros naturais pares parece ser intuitivamente
menordo que N (de fato metadedeste), j a que e subconjunto pr oprio deste.
No entanto, P pode ser posto em correspondencia biunvoca com N: basta
Revista da ORM/SC n
o
3, 2006
Os N umeros e o Innito 55
associar a cada n N o n umero par 2n, isto e, 0 0, 1 2, 2 4, etc.

E
um exerccio simples mostrar que a fun c ao de N em P assim denida e de fato
uma bije c ao (veja o Apendice). Ou seja, (anti-)intuitivamente, isso signica
que P e N tem o mesmo n umero de elementos
7
. Pode-se mostrar [3], mais
geralmente, que uma condi c ao necess aria e suciente para que um conjunto X
seja innito e a existencia de uma bije c ao entre X e um subconjunto pr oprio
de X. De fato, Dedekind usou isso como uma deni c ao alternativa de conjunto
innito.
Outro fato surpreendente: existem tantos n umeros inteiros quanto naturais!
A fun c ao : N Z dada por
(n) =
_
_
_
n
2
se n e par;

n + 1
2
se n e impar.
e uma bije c ao (veja o Apendice para uma prova desse fato). Intuitivamente,
essa fun c ao corresponde a organizar os inteiros na seguinte ordem:
Z = 0, 1, 1, 2, 2, 3, 3, . . . ,
associando o zero ao primeiro da seq uencia, o 1 ao segundo, e assim por diante.
Como se n ao bastasse isso, existem tantos n umeros racionais quantos n umeros
naturais! A demonstra c ao desse fato not avel que apresentamos aqui e um pouco
mais complicada, mas pode ser entendida com alguma paciencia. Come camos
notando que a fun c ao de Z em Q que leva cada inteiro p em
p
1
e injetora. Agora,
lembre que cada racional pode ser escrito, de forma unica, como uma fra c ao
irredutvel
p
q
, onde p, q Z, e podemos assumir, sem perda de generalidade,
que q > 0. Agora denimos uma fun c ao : Q Z assim:
(p/q) =
_
2
p
3
q
se p 0;
2
p
3
q
se p < 0.
Essa fun c ao e injetora (veja Apendice). A seguir, usamos um importante
teorema, cuja demonstra c ao n ao daremos aqui:
7
Observamos, numa nota hist orica, que a descoberta desse fato e atribuda a Galileu
Galilei(1564-1642), que considerou esse absurdouma prova de que n ao poderia haver innito
atual em Matem atica, uma vez que contradizia a m axima aristotelica o todo e maior que as
partes.
Revista da ORM/SC n
o
3, 2006
56 Artigo
Teorema 2 (Schr oder-Bernstein) : Dados conjuntos quaisquer X e Y , su-
ponha que existam fun c oes injetoras f : X Y e g : Y X. Ent ao existe
uma bije c ao entre X e Y .
Demonstra cao: A demonstra c ao pode ser encontrada, por exemplo, em [4].
Aplicando o Teorema de Schr oder-Bernstein ` a nossa situa c ao, conclumos
que existe uma bije c ao entre Z e Q, e portanto h a tantos racionais quanto h a
inteiros. Mas j a vimos que existem tantos inteiros quanto h a naturais.
Deni cao: Um conjunto X e dito ser innito enumer avel, ou simplesmente
enumer avel
8
se, e somente se, existir uma bije c ao entre N e X.
Nesse sentido, o pr oprio N, Z e Q s ao enumer aveis. Um fato importante e
que e possvel mostrar [3] que subconjuntos de conjuntos enumer aveis ou s ao
nitos, ou s ao eles mesmos enumer aveis.
Isso sugere a seguinte pergunta crucial: Ser a que todo conjunto innito e
enumer avel, isto e, ser a que todo conjunto innito tem o mesmo n umero de
elementosde N? A resposta e n ao. H a genuinamente mais n umeros reais do
que n umeros naturais.
Teorema 3 (Cantor) : O conjunto R dos n umeros reais n ao e enumer avel.
Demonstra cao: O metodo usado para esta demonstra c ao e chamado metodo
da diagonal, devido a Cantor. Come camos notando que basta mostrar que
algum subconjunto de R n ao e enumer avel. Se esse for o caso, ent ao o pr oprio R
n ao pode ser enumer avel, pois se fosse, qualquer subconjunto seria enumer avel
(conforme discutido acima) e teramos uma contradi c ao. O subconjunto C que
queremos considerar e o intervalo (0, 1] de todos os n umeros reais r com 0 <
r 1. A demonstra c ao de que C n ao e enumer avel e por contradi c ao. Suponha
que C e enumer avel. Ent ao existe uma bije c ao : N C. Escreva r
n
= (n),
para cada n N. Essa bije c ao nos d a, ent ao, uma listagem C = r
0
, r
1
, r
2
, . . .
de elementos de C. Usamos agora o fato de que cada n umero real r
n
dessa
lista pode ser escrito, de maneira unica, em forma decimal innita sem uma
seq uencia de zeros no m [2]:
r
n
= 0, a
n0
a
n1
a
n2
. . . ,
8
Alguns autores aplicam o adjetivo enumer avel tambem aos conjuntos nitos.
Revista da ORM/SC n
o
3, 2006
Os N umeros e o Innito 57
onde a
ni
0, 1, . . . , 9 para todos n e i. Por exemplo, 0, 7 = 0, 6999 . . . , e
1 = 0, 999 . . . . Considere agora o arranjo duplamente innito
r
0
= 0, a
00
a
01
a
02
. . .
r
1
= 0, a
10
a
11
a
12
. . .
.
.
.
.
.
.
r
n
= 0, a
n0
a
n1
a
n2
. . .
.
.
.
.
.
.
Para cada n, tome b
n
= 1, se a
nn
,= 1 e b
n
= 8, se a
nn
= 1. Ent ao
b = 0, b
0
b
1
. . . b
n
. . . e um n umero real de nosso conjunto C. Logo, b = r
k
,
para algum k. Porem, isso n ao pode ocorrer, uma vez que, da maneira como
obtivemos b, b
k
e certamente diferente de a
kk
. Temos assim a uma contradi c ao;
logo C n ao pode ser enumer avel.
Para o Innito... E Alem!
Cantor generalizou a discuss ao acima para conjuntos quaisquer, introduzindo
a no c ao de cardinalidade, ou tamanho de uma conjunto X, denotada por [X[.
Se X e um conjunto nito, sua cardinalidade e simplesmente seu n umero de
elementos. A deni c ao geral de cardinalidade de um conjunto para incluir con-
juntos innitos e sosticada, e nos levaria muito alem do escopo deste Artigo.
Basta dizer que dois conjuntos X e Y tem a mesma cardinalidade, [X[ = [Y [,
se, somente se, existir uma bije c ao entre X e Y . Intuitivamente, isto signica
que X e Y tem o mesmo n umero de elementos. Assim, N, Z e Q tem a
mesma cardinalidade um do outro, mas diferente da de R.

E possvel ordenar conjuntos por sua cardinalidade. Dados conjuntos X e


Y , diremos que a cardinalidade de X e menor ou igual ` a de Y , [X[ [Y [,
se, e somente se, existir uma fun c ao injetora de X em Y . Intuitivamente, isso
signica que nesse caso X tem o mesmo n umero de elementosque algum
subconjunto de Y , que eventualmente poderia ser todo o Y . O Teorema de
Schr oder-Bernstein (Teorema 2) acima pode ser reinterpretado como dizendo
que se [X[ [Y [ e [Y [ [X[, ent ao [X[ = [Y [. Dizemos que a cardinalidade de
X e (estritamente) menor que a de Y , [X[ < [Y [, se, e somente se, existir uma
fun c ao injetora de X em Y , mas n ao existir uma fun c ao sobrejetora de X em
Y . Ou seja, X tem o mesmo n umero de elementosque algum subconjunto
Revista da ORM/SC n
o
3, 2006
58 Artigo
pr oprio de Y , mas menos elementosque o pr oprio Y . N e estritamente menor
que R nesse sentido
9
. Por isso, apesar de serem ambos conjuntos innitos, R
e de um tipo de innito maiordo que o de N.
Pode-se ainda mostrar os seguintes fatos:
Dados conjuntos X e Y quaisquer, uma e apenas uma das seguintes al-
ternativas ocorre: [X[ < [Y [, [Y [ < [X[, ou [X[ = [Y [. Isto signica
que n umeros innitoss ao ordenados de forma semelhante aos n umeros
usuais;
Se X e um conjunto innito, ent ao [N[ [X[. Ou seja, a cardinalidade
de um conjunto innito enumer avel e a menor possvel entre as cardinal-
idades dos conjuntos innitos. Ou ainda: innito enumer avel e o menor
tipo de innito.
Uma pergunta que se pode fazer agora e: se h a mais de um tipo de innito,
e h a mesmo um menor tipo de innito, ser a que existe o menor tipo de innito?
O seguinte teorema mostra que n ao.
Teorema 4 (Cantor) : Dado uma conjunto qualquer X, denotemos por P(X)
o conjunto das partes de X, isto e, o conjunto de todos os subconjuntos de X.
Ent ao [X[ < [P(X)[.
Esse teorema signica, intuitivamente, que n ao importa qu ao grande seja
o conjunto, seu conjunto das partes e ainda maior. H a portanto toda uma
hierarquia de innitos cada vez maiores, sem m.
Para dar ao leitor uma ideia do impacto hist orico dessas ideias, coletamos
algumas das frases de matem aticos famosos em rela c ao ao tema.
Eu devo protestar veementemente contra o uso do innito como
algo consumado, uma vez que isso nunca e permitido em Matem atica.
C.F. Gauss(1777-1855), matem atico alem ao.
9
Embora exista fun c ao injetora de N em R (por exemplo, a fun c ao que leva cada natural
em sua c opia em R e injetora), pode-se mostrar se existisse uma fun c ao sobrejetora de N em
R, ent ao existiria uma fun c ao injetora de R em N. Mas ent ao, pelo Teorema de Schr oder-
Bernstein, existiria uma bije c ao entre eles, o que n ao pode ocorrer.
Revista da ORM/SC n
o
3, 2006
Os N umeros e o Innito 59
N ao sei o que predomina na teoria de Cantor - Filosoa ou Teolo-
gia, mas estou certo de que n ao h a Matem atica ali.
L. Kronecker(1823-1891), matem atico alem ao.
N ao existe innito atual; os Cantorianos esqueceram-se disto e
caram em contradi c oes. As gera c oes posteriores considerar ao Men-
genlehre [Teoria dos Conjuntos, em alem ao] como uma doen ca da
qual se recobraram.
H. Poincare(1854-1912), matem atico frances.
A teoria de Cantor como um todo e um incidente patol ogico na
hist oria da Matem atica da qual as futuras gera c oes v ao se hor-
rorizar.
L. Brower(1881-1966), matem atico holandes.
A Teoria Axiom atica dos Conjuntos e uma casa construda sobre
areia.
H. Weyl(1885-1955), matem atico alem ao.
Ninguem vai nos tirar do paraso que Cantor criou para n os.
D. Hilbert(1862-1943), matem atico alem ao.
Finalizamos com uma cita c ao do grande matem atico e l ogico polones A.
Tarski(1903-1983), com respeito a um outro tema, mas cuja subst ancia resume
perfeitamente como vemos a no c ao de innito na Matem atica em nossos dias
10
:
[. . . ]A hist oria da ciencia apresenta muitos casos de conceitos que
foram declarados como metafsicos (num sentido indeterminado,
mas em todo caso depreciativo, do termo) antes do seu sentido
ser tornado preciso; contudo, ao receberem uma deni c ao formal
10
Veja [6], pags. 108-109, para o texto integral.
Revista da ORM/SC n
o
3, 2006
60 Artigo
e rigorosa, a desconan ca a seu respeito evaporou-se. Como exem-
plos tpicos, podemos mencionar os conceitos de n umeros negativos
e n umeros imagin arios em Matem atica.[. . . ]parece-me que aqueles
que deles [isto e, dos conceitos] desconaram, com base nas suas ale-
gadas implica c oes metafsicas, devem congratular-se com o fato de
deni c oes precisas de tais conceitos estarem agora disponveis. Se,
em conseq uencia, os conceitos [. . . ] perderem interesse los oco,
ent ao eles apenas partilhar ao o destino de muitos outros conceitos
cientcos, e n ao h a que lamentar tal fato.
Apendice: Elementos de Fun c oes
Tratamos aqui de alguns aspectos b asicos acerca de fun c oes. Para uma
introdu c ao elementar, porem sistem atica, veja [5].
Dados conjuntos X e Y quaisquer, uma fun c ao (ou aplica c ao) f de X em
Y , denotada freq uentemente por f : X Y , e uma regra
11
que, para todo
elemento x de X, associa um unico elemento f(x) de Y , chamado o valor da
fun c ao f em (ou no argumento) x. Nesse caso, X e dito ser o domnio da fun c ao
f e Y e o contradomnio de f.
Uma fun c ao f : X Y e dita ser injetora (ou injetiva) se, e somente
se, dados elementos quaisquer x, x

X, se f(x) = f(x

), ent ao x = x

, ou
equivalentemente, se x ,= x

, ent ao f(x) ,= f(x

). Em outras palavras, uma


fun c ao e injetora se n ao assume o mesmo valor em dois argumentos distintos.
A fun c ao g : R R dada por g(x) = x
2
n ao e injetora, pois temos, por
exemplo, g(2) = g(2) = 4.
Vejamos alguns exemplos do texto principal:
A fun c ao f : N P que a cada n N, associa o n umero par 2n e injetora,
pois para quaisquer n, n

N, f(n) = f(n

) 2n = 2n

n = n

.
A fun c ao : N Z tambem e injetora. De fato, dados n, n

N, se
(n) = (n

), ent ao n e n

ou s ao ambos pares ou ambos mpares. No primeiro


caso,
n
2
=
n

2
n = n

. O outro caso e an alogo.


: Q Z e tambem injetora. De fato, dados
p
q
,
p

Q, se (
p
q
) = (
p

),
11
O leitor mais exigente notar a que, embora intuitivamente ntida, h a certa imprecis ao
nessa deni c ao; por exemplo, o que e exatamente uma regra?

E possvel dar uma deni c ao
mais precisa de fun c ao, porem isso nos desviaria demais dos ns deste Artigo.
Revista da ORM/SC n
o
3, 2006
Os N umeros e o Innito 61
ent ao necessariamente, p e p

tem o mesmo sinal. Suponha que sejam ambos


n ao-negativos. Se ambos forem negativos, o argumento e an alogo. Temos ent ao
2
p
3
q
= 2
p

3
q

. Mas a decomposi c ao em fatores primos de um mesmo n umero e


unica (Teorema Fundamental da Aritmetica). Portanto p = p

e q = q

.
Dada uma fun c ao f : X Y , a imagem de f, denotada Imf, e o sub-
conjunto de Y que s ao valores de f. Em outras palavras, s ao aqueles y Y
para os quais existe x X tal que f(x) = y. Claro que Imf Y mas em
geral Imf ,= Y . Se Imf = Y , f e dita ser sobrejetora (ou sobrejetiva). A
fun c ao : Q Z acima, por exemplo, n ao e sobrejetora, j a que qualquer
n umero primo maior que tres em Z j a n ao estar a em sua imagem. J a a fun c ao
f : N P acima e claramente sobrejetora.
Se uma fun c ao e sobrejetora e injetora simultaneamente, ent ao a mesma
diz-se ser bijetora, ou uma bije c ao, ou ainda, uma correspondencia biunvoca.
A fun c ao : N Z acima e uma bije c ao. Com efeito, j a vimos ser ela injetora.
Para ver que e sobrejetora, tome um a Z arbitr ario. Vamos mostrar que
existe algum n
a
N tal que (n
a
) = a. De fato, se a 0, tome n
a
= 2a, e se
a < 0, tome n
a
= 2(a) 1. O leitor pode checar ent ao, usando a deni c ao de
do texto, que (n
a
) = a. Mas como a e arbitr ario, todo a Z e valor de ,
que e portanto uma bije c ao.
REFER

ENCIAS
[1] C.B. Boyer, Hist oria da Matem atica, 2
a
Edi c ao, Ed. Edgar Bl ucher,
S ao Paulo, 1996. V.J. Katz, A History of Mathematics - An Introduction, 2
a
Edi c ao, Ed. Addison Wesley Longman, Reading, 1998.
[2] H.H. Domingues, Fundamentos de Aritmetica, Atual Editora, S ao Paulo,
1991.
[3] E.L. Lima, Curso de An alise - Vol.1, 6
a
Edi c ao, Projeto Euclides, IMPA,
Rio de Janeiro, 1989.
[4] M. Aigner e G.M. Ziegler, As Provas Est ao nO LIVRO, Ed. Edgar
Bl ucher, S ao Paulo, 2002.
Revista da ORM/SC n
o
3, 2006
62 Artigo
[5] G. Iezzi e C. Murakami, Fundamentos de Matem atica Elementar, vol.1,
7
a
Edi c ao, Atual Editora, S ao Paulo, 1996
[6] A. Tarski, A Concep c ao Sem antica da Verdade e os Fundamentos da
Sem antica, Existencia e Linguagem - Ensaios de Metafsica Analtica (Ed. J.
Branquinho), Editorial Presen ca, Lisboa, 1990.
Revista da ORM/SC n
o
3, 2006
Artigo
64 Artigo
Revista da ORM/SC n
o
3, 2006
Problemas Olmpicos 65
Problemas Olmpicos: Analise quanto `as
Diferentes Tecnicas de Resolu cao
Juliana Duarte Zacchi
juzacchi@yahoo.com.br
Para muitos educadores a resolu c ao de problemas e uma grande priori-
dade no ensino de matem atica. No entanto, professores de ciencias exatas de
faculdades s ao testemunhas da diculdade encontrada pelos seus alunos na re
solu c ao de problemas, o quanto se sentem perdidos ao se depararem com um
problema n ao rotineiro.
Nos ensinos Fundamental e Medio, a enfase no estudo de matem atica e
na aprendizagem e aplica c ao de algoritmos envolvendo c alculos, o que, muitas
vezes, torna o estudo enfadonho.
H a uma forte tendencia hoje em dia em contextualizar a matem atica, para
que o aluno perceba as v arias aplica c oes do conte udo que aprendeu. Mas,
devido a complexidade dessas aplica c oes, essa ideia rapidamente deixa de ser
pratic avel em sala de aula.
Trabalhar com resolu c ao de problemas n ao rotineiros em sala de aula, con-
tribui para o desenvolvimento da inteligencia, pois o aluno e estimulado a en-
contrar um caminho n ao conhecido de antem ao (contornando obst aculos) para
alcan car um m desejado.

E muito comum nos depararmos com algumas teorias da did atica, ou metodos
pedag ogicos que, apesar de nos parecer propcios, n ao s ao pratic aveis. O que
os professores mais alegam e que: Isso toma tempo demais; s o tenho 3 aulas
semanais, n ao conseguirei cumprir o programa, etc.
Por essas raz oes acredita-se que a Olimpada Regional de Matem atica de
Santa Catarina (bem como as outras competi c oes a nvel nacional ou interna-
cional) possa ser um forte instrumento para o professor trabalhar com resolu c ao
de problemas com seus alunos. Uma das vantagens desta competi c ao e que e
uma atividade extra-classe em que o professor n ao precisa se preocupar com a
elabora c ao dos problemas.
Revista da ORM/SC n
o
3, 2006
66 Artigo
Os problemas olmpicos exigem muita criatividade e imagina c ao em suas
resolu c oes. Mais do que um malabarismo com contas, eles exigem ideias
muitas vezes simples mas brilhantes.
S ao problemas n ao convencionais, pois exigem pouco uso de f ormulas e n ao
s ao habitualmente encontrados nos livros did aticos.
Arma c oes como estas s ao geralmente encontradas quando faz-se referencia
aos problemas olmpicos. Mas, anal:
O que caracteriza um problema olmpico?
No que eles se diferem dos problemas usuais?
Am de tentar responder ` a essas quest oes faz-se aqui uma an alise destes
problemas quanto ` as diferentes tecnicas de resolu c ao. Esta an alise concentra-se
nos problemas olmpicos da segunda fase da ORM e baseia-se em uma teoria
da did atica da matem atica de Ives Chevallard.
Uma das preocupa c oes da comiss ao que elabora estes problemas, e que o
conte udo exigido para sua resolu c ao esteja adequado ao nvel da quest ao. Um
problema de nvel 2, por exemplo, deve abordar somente conte udos vistos ate
a 8
a
serie.
Os problemas olmpicos, portanto, est ao vinculados aos conte udos dos livros
did aticos, porem n ao vivem neles. Dicilmente s ao encontrados em livros
did aticos problemas no estilo olmpico.
Quadro Te orico
A an alise dos problemas olmpicos utilizar a conceitos da Teoria Antropo-
l ogica do Saber de Ives Chevallard.
Chevallard se utiliza metaforicamente de termos ecol ogicos como habitat e
nicho am de ilustrar que um saber n ao vive isolado, ele est a ligado a insti-
tui c oes (habitats) e desempenha uma fun c ao (nicho).
Aqui o signicado de institui c ao n ao est a necessariamente atrelado a insti-
tui c oes de ensino, e mais amplo, pode ser, por exemplo, um livro did atico ou
um artigo de revista. Neste caso, o nosso objeto (saber) matem atico s ao os
problemas olmpicos e tem como institui c ao as Olimpadas de Matem atica.
A descri c ao que Chevallard faz de uma organiza c ao matem atica ser a a base
para a an alise dos problemas olmpicos. Ele as descreve em termos de tarefa,
tecnica, tecnologia e teoria relativas a um objeto matem atico.
Revista da ORM/SC n
o
3, 2006
Problemas Olmpicos 67
Em rela c ao ao objeto matem atico estudado (os problemas olmpicos), temos:
Tarefa - e uma a c ao, e a pergunta do problema, por exemplo: calcular,
determinar, demonstrar.
Tecnica - o que e feito para realizar a tarefa, a maneira com que se chega
` a solu c ao do problema (o que foi utilizado). Por exemplo: algoritmos,
experimenta c oes.
Obs: e o estudo da tecnica que nos permitir a classicar os problemas em
tipos ou classes.
Tecnologia - e o que valida a tecnica, e o suporte te orico, o que est a por
tr as. Por exemplo: deni c oes, teoremas, propriedades.
Teoria - e a tecnologia de uma tecnologia, um discurso mais amplo que
justique a tecnologia. Exemplo: algebra, geometria, teoria de conjuntos.
Para esta an alise foi escolhida aqui uma prova da segunda fase da ORM
de SC. A prova e do nvel 1 do ano de 2001. Para classicar os problemas em
termos de tarefa, tecnica, tecnologia e teoria, primeiramente apresentamos os
enunciados e solu c oes destes.
Listagem dos problemas com resolu c ao:
1. Um trabalhador limpa um terreno em quatro horas e outro trabalhador
limpa o mesmo terreno em oito horas. Quanto tempo os dois trabalha-
dores, trabalhando juntos, levam para limpar o terreno? Dar a resposta
em horas e minutos.
Resolu c ao: O trabalhador que limpa o terreno todo em 4 horas e duas
vezes mais r apido que o trabalhador que limpa o mesmo terreno em 8 ho-
ras. Assim, em um mesmo intervalo de tempo, aquele trabalhador limpa
o dobro do terreno que este ultimo trabalhador. Portanto, o trabalhador
mais r apido limpa
2
3
do terreno (gastando
2
3
4 =
8
3
de hora), enquanto
que o outro limpa
1
3
do mesmo terreno para a tarefa estar terminada
(este trabalhador gasta os mesmos
8
3
de hora,
1
3
8 =
8
3
).
Revista da ORM/SC n
o
3, 2006
68 Artigo
Como 1 h = 60 min e
1
3
h = 20 min, ent ao
8
3
h = 8 20 = 160 min =
120 min + 40 min = 2 h e 40 min.
Portanto, os dois trabalhadores levam 2 h e 40 min para limparem o ter-
reno.
2. Com os algarismos 1, 2, 3, 4, 5, 6 e 7 deseja-se construir n umeros de tres
algarismos satisfazendo as seguintes condi c oes:
(a) Em cada n umero os algarismos devem estar em ordem crescente da
esquerda para a direita, ou seja, o algarismo da centena deve ser
menor do que o algarismo da dezena, e este deve ser menor do que
o algarismo da unidade.
(b) Para quaisquer dois algarismos considerados (ou seja, dentre os sete
algarismos acima) deve existir sempre um unico n umero onde estes
algarismos aparecem ao mesmo tempo (exemplo: dados os algaris-
mos 1 e 2, devemos ter um e somente um dos n umeros 123, 124, 125,
126 ou 127).
Quantos n umeros e preciso construir de modo que as duas condi c oes sejam
satisfeitas? Apresente estes n umeros.
Resolu c ao: Vamos come car construindo os n umeros 123, 145 e 167 satis-
fazendo as condi c oes (a) e (b) para qualquer par que contenha o algarismo
1. A condi c ao (b) ainda n ao est a completamente satisfeita.
J a existe um n umero com os algarismos 1 e 2, e com 2 e 3 (o n umero
123), mas n ao existe ainda nenhum n umero com os algarismos 2 e 4 ou 2
e 5 ou 2 e 6 ou 2 e 7.
Tentamos ent ao 246 e 257 e esgotamos a condi c ao (b) em rela c ao ao al-
garismo 2. Falta ainda vericar esta condi c ao para os algarismos 3, 4, 5,
6 e 7.
Vejamos: 347 e 356.
Podemos vericar que estes 7 n umeros satisfazem as condi c oes (a) e (b):
123 246 347
145 257 356
167
Revista da ORM/SC n
o
3, 2006
Problemas Olmpicos 69
Obs:
Existem outras solu c oes possveis. Por exemplo:
123 247 346
145 256 357
167
Estas s ao as duas unicas solu c oes possveis, xados os n umeros 123,
145 e 167.Note que, xado o n umero 123, temos 3 possibilidades
para os outros n umeros com o algarismo 1: 145 (e da 167), 146 (e
157) e 147 (e 156). Finalmente, com os algarismos 1 e 2 temos 5
possibilidades: 123, 124, 125, 126 e 127. Portanto o n umero total
de solu c oes possveis e: 5 3 2 = 30.
O n umero 7 pode ser obtido da seguinte maneira: com 7 algarismos
podemos formar C
3
7
= 35 n umeros (que ter ao seus algarismos em
ordem crescente) distintos de 3 algarismos. Para cada par de al-
garismos existem 5 possibilidades de formar um n umero. Portanto,
35 5 = 7.
3. Usando moedas nos valores de 1, 5, 10, 25 e 50 centavos, qual o menor
n umero de moedas necess ario para pagar uma conta de 94 centavos? E
se a conta for de 99 centavos?
Resolu c ao:Para formar 94 centavos necessitaremos de 4 moedas de 1 cen-
tavo (n ao h a outra possibilidade). Faltam 90 centavos.
Para formar 90 centavos com o menor n umero possvel de moedas, parti-
mos das maiores moedas:
50 + 25 +. . . ?
E a necessitamos de pelo menos uma moeda de 5 centavos.
Assim teremos:
90 = 50 + 25 + 5 + 10
Qualquer outra possibilidade necessitar a mais moedas:
90 = 50 + 10 + 10 + 10 + 10 = 50 + 25 + 5 + 5 + 5
Revista da ORM/SC n
o
3, 2006
70 Artigo
etc. Portanto, o menor n umero para pagar a conta de 94 centavos e 8:
94 = 1 + 1 + 1 + 1 + 50 + 25 + 5 + 10
Se a conta for de 99 centavos, ainda necessitamos de 8 moedas:
99 = 1 + 1 + 1 + 1 + 25 + 50 + 10 + 10
4. Observe as igualdades:
2
2
1
2
= 3
3
2
2
2
= 5
4
2
3
2
= 7
Explique porque a diferen ca entre os quadrados de dois n umeros consec-
utivos e sempre um n umero mpar.
Resolu c ao: Se considerarmos dois n umeros consecutivos, um deles ser a
par e o outro mpar. O quadrado de um n umero par e par e o quadrado
de um n umero mpar e mpar. Assim, a diferen ca entre um n umero par
e um n umero mpar (ou mpar e par) ser a mpar.
Algebricamente:
(n + 1)
2
n
2
= 2n 1 (mpar).
5. Uma sala de oitos metros por quatro metros deve ser ladrilhada com
ladrilhos quadrados de 25 cm de lado. O dono da sala resolve usar
ladrilhos vermelhos e azuis, formando blocos de mesma cor no seguinte
formato:
Ele tambem quer que a sala seja ladrilhada com estes blocos de forma que
os blocos da mesma cor n ao quem encostados lado a lado (e permitido
que se encostem em um vertice). Ser a possvel ladrilhar a sala com estas
Revista da ORM/SC n
o
3, 2006
Problemas Olmpicos 71
exigencias usando um n umero inteiro de ladrilhos de cada cor? Justique
sua resposta.
Resolu c ao: Podemos formar blocos de 1m x 1m com 4 blocos b asicos
(dados no enunciado) da seguinte maneira:
A
A A A
A
A A A
V
V V V
V
V V V
onde A = azul e V = vermelho.
Agora basta juntar 8 colunas, cada uma com 4 blocos destes acima.
Teremos portanto: 8 4 = 32 blocos iguais aos blocos acima, ou seja,
32 4 = 128 blocos b asicos, sendo 64 vermelhos e 64 azuis. Teremos
portanto: 4 64 = 256 ladrilhos azuis e 256 ladrilhos vermelhos.
Classica cao
Para cada problema apresentado acima faz-se uma classica c ao em termos
de tarefa, tecnica, tecnologia e teoria (segundo conceitos de Ives Chevallard).
ORM - 2001, Nvel 1: Problema 1
i) Tarefa: determinar quanto tempo levam os dois trabalhadores para
limparem o terreno;
ii) Tecnica: opera c oes; reconhecimento do dobro de um n umero; repre-
senta c ao decimal;
iii) Tecnologia: algoritmo das opera c oes; sistema de numera c ao; sistema de
medidas;
iv) Teoria: aritmetica.
Revista da ORM/SC n
o
3, 2006
72 Artigo
ORM - 2001, Nvel 1: Problema 2
i) Tarefa: determinar quantos n umeros s ao possveis de se construir satis-
fazendo as condi c oes propostas e apresent a-los;
ii) Tecnica: opera c oes; arvore;
iii) Tecnologia: algoritmo das opera c oes; rela c ao de ordem; sistema de nu-
mera c ao;
iv) Teoria: aritmetica; an alise combinat oria.
ORM - 2001, Nvel 1: Problema 3
i) Tarefa: determinar o menor n umero de moedas para pagar a conta;
ii) Tecnica: opera c oes; parti c ao de um n umero;
iii) Tecnologia: algoritmo das opera c oes;
iv) Teoria: aritmetica.
ORM - 2001, Nvel 1: Problema 4
i) Tarefa: explicar porque a diferen ca entre os quadrados de dois n umeros
consecutivos e sempre um n umero mpar;
ii) Tecnica: opera c oes; reconhecimento de padr oes; reconhecimento de n umeros
pares; reconhecimento de n umeros mpares;
iii) Tecnologia: algoritmo das opera c oes;
iv) Teoria: aritmetica.
ORM - 2001, Nvel 1: Problema 5
i) Tarefa: Determinar se e possvel ladrilhar a sala com as exigencias do
dono;
ii) Tecnica: reconhecimento de padr oes; pavimenta c ao do plano;
Revista da ORM/SC n
o
3, 2006
Problemas Olmpicos 73
iii) Tecnologia: algoritmo das opera c oes;
iv) Teoria: aritmetica; geometria.
Abaixo s ao descritas as tecnicas que guraram dentre os problemas listados
anteriormente.
Opera c oes: e a tecnica mais elementar. Todos os problemas analisados
exigem essa tecnica que consiste no uso das quatro opera c oes b asicas
(adi c ao, subtra c ao, multiplica c ao e divis ao);
Reconhecimento do dobro de um n umero: capacidade de reconhecer quando
um n umero e o dobro de outro;
Reconhecimento de n umeros pares e n umeros mpares: capacidade de
identicar se um n umero e par ou mpar;
Reconhecer um n umero par, por exemplo, exige do aluno uma boa com-
preens ao da deni c ao de n umeros pares. Todas as tecnicas desse tipo exigem
a compreens ao da teoria envolvida.
Reconhecimento de padr oes: geralmente o aluno percebe um padr ao em
seus c alculos quando h a uma repeti c ao constante de resultados;


Arvore: tecnica bastante utilizada para listar todas as possibilidades para
um certo evento.

E uma maneira sistem atica de list a-las sem repetir
qualquer possibilidade ou esquecer de alguma;
Parti c ao de um n umero: decompor um n umero natural como soma de
naturais.
Pavimenta c ao do plano: dispor guras geometricas de modo a cobrir uma
regi ao do plano sem que haja superposi c ao ou espa cos vazios.
Algumas tecnicas utilizadas na resolu c ao dos problemas olmpicos tambem
s ao freq uentemente encontradas nos livros did aticos como, por exemplo, a re-
presenta c ao decimal.
Em um estudo mais aprofundado desses problemas (ver [1]) tecnicas como
a experimenta c ao e reconhecimento de padr oes s ao encontradas com bastante
Revista da ORM/SC n
o
3, 2006
74 Artigo
freq uencia o que talvez possa responder em parte as perguntas do incio deste
artigo: O que caracteriza um problema olmpico? No que eles se diferem dos
problemas usuais?
Essas tecnicas s ao raramente encontradas na resolu c ao de exerccios de livros
did aticos, o que torna os problemas olmpicos diferentes. S ao tecnicas como
estas que despertam o raciocnio l ogico nos alunos, o que vem a ser uma das
caractersticas destes problemas.
As quest oes discursivas exigem que o aluno exponha seu raciocnio de forma
clara e compreensvel, o que permite a eles uma sistematiza c ao do raciocnio e
uma melhor compreens ao da solu c ao apresentada.
Os problemas olmpicos s ao formulados de modo que n ao apresente t ao
claramente os conceitos matem aticos utilizados em sua resolu c ao, levando o
aluno a relacionar as f ormulas, teoremas e resultados que aprende na sala de
aula a estas situa c oes-problema.
Criatividade e originalidade s ao necess arias para resolver estes problemas,
pois os mesmos geralmente descrevem situa c oes para as quais nenhum processo
rotineiro foi previamente aprendido. Ao resolver estes problemas com certa
freq uencia, o aluno adquire suas mais variadas tecnicas, ampliando assim seu
conhecimento matem atico.
Esta an alise feita com uma prova de nvel 1 revela que a teoria envolvida
nos problemas e essencialmente a aritmetica (com algumas no c oes b asicas de
geometria e an alise combinat oria), visto que alunos 5
a
e 6
a
series ainda n ao
conhecem a algebra e pouco sabem sobre geometria, por exemplo.
Percebe-se tambem que as tecnologias envolvidas nesses problemas s ao bas-
tante elementares, n ao h a nada muito sosticado, o que nos faz crer que a
diculdade encontrada em suas resolu c oes consiste mais em interpretar o enun-
ciado e ter uma ideia inicial da resolu c ao.
Julgando de suma import ancia a resolu c ao de problemas para o desenvolvi-
mento do pensamento l ogico e raciocnio crtico dos alunos, a an alise apre-
sentada aqui, utilizando conceitos da Teoria Antropol ogica do Saber de Ives
Chevallard, pode em muito auxiliar o professor na hora de escolher os proble-
mas a serem trabalhados com os alunos em sala de aula.
Revista da ORM/SC n
o
3, 2006
Problemas Olmpicos 75
REFER

ENCIAS
[1] ZACCHI, Juliana Duarte. Problemas Olmpicos. 68s. 2004 (Trabalho
de Conclus ao de Curso) Curso de Licenciatura em Matem atica, Departamento
de Matem atica, Universidade Federal de Santa Catarina.
Revista da ORM/SC n
o
3, 2006
76 Artigo
Revista da ORM/SC n
o
3, 2006
Artigo
78 Artigo
Revista da ORM/SC n
o
3, 2006
O Problema da Divis ao da Pizza 79
O Problema da Divisao da Pizza
William Glenn Whitley
Departamento de Matem atica, UFSC (Aposentado)
A Professora Carmem Suzane Comitre Gimenez apresentou um problema
interessante sobre como cortar uma pizza e distribuir as fatias (Revista da
Olimpada Regional de Matem atica de Santa Catarina - N
o
2, pg 116, Problema
5).
Consideramos uma pizza em forma de um disco circular perfeito com cen-
tro O na origem de um sistema de coordenadas cartesianas. Consideramos,
tambem, que a pizza e cortada em oito fatias. Concordamos que todos os
cortes passam por um ponto que chamaremos de V e que os angulos dos bicos
das fatias em V s ao todos de 45
o
. Armamos que se voce d a peda cos alterna-
dos para duas pessoas, cada uma receber a 4 peda cos e a quantia total de pizza
dada a cada pessoa e a mesma. Veja a Figura 1.
Figura 1
Na Figura 1, marcamos os quatros cortes com estilos de linhas diferentes,
etiquetamos os pontos onde os cortes (cordas) interceptam a circunferencia e
indicamos a distribui c ao das fatias para a pessoa 1 ou a pessoa 2. Nesta gura
notamos que o ponto V est a bem afastado do centro O da circunferencia. Se
Revista da ORM/SC n
o
3, 2006
80 Artigo
um dos cortes passa por O, a solu c ao e imediata, n ao sendo discutida aqui.
Portanto, podemos girar e/ou reetir o desenho ate o ponto V estar no primeiro
quadrante e a corda mais curta ser horizontal.
Na Figura 2, as quatro cordas iniciais s ao salientadas pelas retas mais
grossas, deixando mais clara a divis ao do disco nos peda cos de pizza. Em
seguida, incluimos c opias destas cordas obtidas atraves de rota c oes de 90
o
,
180
o
e 270
o
das cordas originais. Queremos usar as simetrias da gura resul-
tante para apontar pedacinhos menores de areas iguais nas diferentes fatias.
Por ultimo, nas guras futuras, as fatias que v ao para a pessoa 1 ganhar ao um
sombreamento cinza, enquanto as da pessoa 2 permanecer ao brancas.
Figura 2
Antes de iniciar esta tarefa, devemos esclarecer um ponto de terminolo-
gia. Rota c ao de 90
o
, reex ao vertical e reex ao horizontal s ao termos bastante
claros, mas reex ao em 45
o
e um pouco ambguo. Anal, h a dois eixos di-
agonais. Chamaremos a diagonal que desce enquanto progride para a direita
(inclina c ao 1 no sistema de coordenadas cartesianas) de diagonal negativa e o
outro (com inclina c ao 1) de positiva. Chamamos a aten c ao para uma situa c ao
especial neste desenho. Uma corda, aquela representada pela reta s olida, e
curta o suciente para que o quadrado formado por suas imagens tenha seus
vertices fora da circunferencia. O quadrado formado pelas cordas pontilhadas
tem seus vertices na borda e os outros dois quadrados tem seus vertices no
interior do disco. Dependendo de quanto o ponto de intersec c ao das cordas e
distante do centro, esta situa c ao pode se alterar signicativamente. Portanto,
enquanto a distribui c ao das fatias n ao se altera, a congura c ao dos pedacinhos
Revista da ORM/SC n
o
3, 2006
O Problema da Divis ao da Pizza 81
do nosso desenho podem se alterar nos cantos perto dos di ametros dos quadra-
dos. Tentaremos fazer um agrupamento de peda cos que n ao dependa desta
propriedade especial do desenho.
Na Figura 3 vemos as quatro fatias menores acima da corda s olida, cada
uma com marca c ao diferente. Em seguida vemos c opias de duas delas, as duas
mais para a esquerda, sob a reex ao pela diagonal positiva e c opias das outras
duas sob a reex ao pela diagonal negativa. Como reex ao preserva area, para
cada uma destas quatro fatias, achamos uma area equivalente, sempre dentro
de uma fatia no outro agrupamento.
Neste momento as quatro fatias menores delimitadas pela corda s olida j a
foram compensadas por partes de outras fatias. Das fatias menores delimitadas
pela corda pontilhada, tres j a foram compensadas, e uma parte da quarta fatia
foi consumida. Desejamos compensar o resto desta fatia. Para facilitar os de-
senhos futuros, anotaremos as areas j a compensadas preenchendo-as com feixes
de linhas horizontais. Novas areas em discuss ao ser ao marcadas preferencial-
mente com feixes de linhas diagonais.
Figura 3
Na busca de um lugar para encaixar esta regi ao, giramos e reetimos a
gura ` a procura de uma regi ao sombreada de formato igual. Achamos duas.
Podemos reetir em rela c ao ao eixo vertical ou girar 90
o
no sentido hor ario.
Optamos por girar. O que aconteceria se voce optasse para reetir? Daria para
compensar as regi oes restantes, fornecendo outra solu c ao? Veja a Figura 4.
Passamos a examinar a fatia grande delimitada pelas cordas pontilhada
e tracejada. Notamos que e uma regi ao quadril atera e que os lados opostos
Revista da ORM/SC n
o
3, 2006
82 Artigo
formados por partes das cordas s olida e tracejada s ao paralelas. Temos assim
um trapezio T que e muito especial, seus angulos medem ou 45
o
ou 135
o
.
Figura 4
Se examinamos o tri angulo XZV notamos que os angulos em X e Z s ao
de 45
o
e o tri angulo e is osceles. Assim, a corda tracejada que passa por V e
perpendicular ` a base e e a sua mediatriz, ou seja, XY = Y Z. Devido ` a simetria
por reex ao vertical, XX

e vertical e V XX

e um trapezio com as mesmas


bases que T e a mesma altura. Portanto, eles tem a mesma area. Deste modo,
identicamos dois pares de areas que podem ser compensadas.
Figura 5
Revista da ORM/SC n
o
3, 2006
O Problema da Divis ao da Pizza 83
Se pretendemos continuar nossa tecnica, tentaremos achar um local para
encaixar o que sobrou da fatia grande tracejada simples-tracejada mista. Infe-
lizmente, a regi ao que sobrou e toda irregular e temos somente um lugar para
encaix a-la, a fatia tracejada mista-s olida. Notamos que a regi ao n ao compen-
sada delimitada pelas cordas s olida e a tracejada mista e uma regi ao convexa.
Podemos tentar cortar fora um bico da regi ao que sobra na fatia tracejada
simples-tracejada mista na esperan ca de encaixar um grande peda co convexo e
esperar pela inspira c ao para acertar os peda cos menores que sobram. Na Figura
6 identicamos uma regi ao promissora e a giramos 90
o
no sentido hor ario.
Notamos que sobrou uma regi ao triangular em cada fatia. Armamos que
estes tri angulos s ao tri angulos ret angulos is osceles. Notamos tambem que as
suas hipotenusas tem o mesmo comprimento por ambos medirem a dist ancia
entre um par de cordas tracejadas paralelas. Portanto, os tri angulos s ao con-
gruentes e tem a mesma area.
Isto conclui a demonstra c ao.
Na realidade, ainda n ao conclu. Lembram como colocamos a corda menor
em cima do desenho e com o vertice dos cortes mais para a direita? Onde
usamos este fato na demonstra c ao? Ser a que n ao foi necess ario ou ser a que
enganamos voces e usamos sem avisar?
Figura 6
Ser a que esta e a unica solu c ao? Podemos achar outras maneiras de recortar
as fatias ou ate outras maneiras de agrupar os pedacinhos do nosso desenho?
Quantas outras simetrias podem identicar na Figura 2 alem daquelas usadas
aqui? Divirtam-se.
Revista da ORM/SC n
o
3, 2006
84 Artigo
Revista da ORM/SC n
o
3, 2006
Artigo
86 Artigo
Revista da ORM/SC n
o
3, 2006
Se a Terra n ao e Plana... 87
Se a Terra nao e Plana, quais sao as Rela c oes
Metricas adequadas para determinarmos
Comprimentos e

Angulos?
Celso Melchiades Doria
Dep. de Matem atica, Universidade Federal de Santa Catarina
CEP: 88.040-900, Florian opolis-SC
O pregador h a de ser como quem semeia, e n ao como quem ladrilha ou
azuleja . . . O r ustico acha documentos nas estrelas para a sua lavoura e o
mareante para a sua navega c ao e o matem atico para as suas observa c oes e
para os seus juzos. De maneira que o r ustico e o mareante, que n ao sabem
ler nem escrever, entendam as estrelas; e o matem atico, que tem lido quantos
escreveram, n ao alcan ca a entender quanto nelas h a.- P
e
. Ant onio
Vieira(1608-1697) - Serm ao da Sexagesima [5].
Introdu cao
Se a primeira impress ao foi de que a Terra era plana, nada mais natural do
que descobrir, primeiramente, que num tri angulo ret angulo ABC (g.1) cuja
hipotenusa mede a e os catetos medem b e c, que o quadrado da hipotenusa e
igual a soma do quadrado dos catetos , ou seja,
a
2
= b
2
+c
2
. (1)
A identidade 1 e a express ao do famoso Teorema de Pit agoras.
Decorre do Teorema de Pit agoras que num tri angulo ABC qualquer, cujos
lados medem a, b e c, e cujos angulos internos medem , e , conforme indica
a gura 2, que
Revista da ORM/SC n
o
3, 2006
88 Artigo
a
2
=b
2
+c
2
2bc.cos()
b
2
=a
2
+c
2
2ac.cos(),
c
2
=a
2
+b
2
2ab.cos().
(2)
a
b
c

A B
C
Figura 1: tri angulo ret angulo
A
B
C
b
a
c

Figura 2: tri angulo ABC


Porem, a Terra n ao sendo plana implica que as identidades acima n ao s ao as
mais adequadas para realizarmos medi c oes sobre a superfcie do nosso planeta.

E claro, a experiencia mostra que quando as medidas realizadas s ao pequenas


em rela c ao ao raio da Terra, ent ao os resultados obtidos pelas identidades 1 e
2 s ao bastante precisos.

E a que come cam os problemas, uma vez que a neces-
sidade de realizarem-se medidas de longa dist ancias sobre a Terra e inevit avel.
O objetivo deste artigo e descrever as rela c oes metricas em tri angulos esfericos
e mostrar um metodo de obte-las. Se assumirmos que a Terra e uma esfera, o
principal par ametro a ser determinado e a medida do seu raio.
Raio da Terra
Medida do Raio da Terra
Por volta de 250 a.c., o grego Erat ostenes (276 - 194 a.c.), amigo de Ar-
quimedes e conhecido como Beta, por ser o segundo melhor em tudo, desen-
volveu um metodo muito simples para calcular a medida da circunferencia da
Terra. Hoje em dia sabemos que e de 40.075 km. Considerando que na epoca
Revista da ORM/SC n
o
3, 2006
Se a Terra n ao e Plana... 89
n ao se sabia qual era o formato da Terra, podemos comparar o c alculo de
Erat ostenes ` a obten c ao de uma resposta para a quest ao atual sobre o formato
do Universo (espa co-tempo).
Erat ostenes exercia o cargo de administrador da Biblioteca de Alexandria,
no Egito, onde havia v arios pergaminhos com conhecimentos diversos, dentre
os quais os adquiridos pelos Gregos e pelos Egpcios. A aproximadamente 800
km ao sul de Alexandria havia uma cidade, denominada na epoca de Syene e
hoje conhecida como Aswan, onde Erat ostenes sabia que a posi c ao do Sol, ao
atingir o zenite no solstcio, era vertical.

A
B
SOL

O
TERRA
Figura 3: Alexandria e Seyne , =
Na g.3, o ponto A corresponde ` a cidade de Syene enquanto o ponto B
` a Alexandria. Erat ostenes concluiu que lhe bastava conhecer o angulo e
a dist ancia Aswan-Alexandria para estimar a circunferencia da Terra. Isto
porque ele conhecia as f ormulas AB = R. R =
AB

e C = 2.R, da onde
C = 2.
AB

. (3)
Estas f ormulas, consideradas evidentes nos dias de hoje, eram grosseira-
mente deduzidas e n ao dispunham de uma representa c ao algebrica adequada
para manipul a-las, tornando o conhecimento e as aplica c oes acessveis para
poucos.
A ideia de Erat ostenes foi determinar o angulo , o que ele fez considerando
as seguintes hip oteses:
1. A Terra e uma Esfera, assim como a Lua.
2. Os raios do sol chegam ` a Terra praticamente paralelos;
Revista da ORM/SC n
o
3, 2006
90 Artigo
3. O caminho que percorre a menor dist ancia entre Alexandria e Aswan,
se continuado, descreve uma circunferencia igual a de um grande crculo
(Equador)
Desta forma, Erat ostenes simplicou o problema da determina c ao do angulo
. No instante em que o raio de Sol atinge o ponto A ortogonalmente, o
mesmo raio ao atingir o ponto B forma um angulo com uma estaca ncada
ortogonalmente ao ch ao. Na gura, observamos que os angulos e s ao iguais,
uma vez que angulos opostos pelo vertice s ao congruentes, assim como angulos
correspondentes tambem s ao.
Erat ostenes mediu =

25
e

AB = 5.000 est adios
12
, da onde C = 2
25 5000 = 250.000 est adios. Considerando que 1 est adio correspondia a 157,5
metros, segue que C = 39.375 Km e R = 6.266, 71 km. Apesar do metodo
utilizado ser pouco preciso, o resultado obtido e excelente pois ao compar a-lo
com a medida atual de R = 6.378, 11 km, obtido a partir dos 40.075 km de
circunferencia, o erro e da ordem de 111 km (1, 75%).
Se tal extraordin ario resultado pode ser feito conhecendo-se ape-
nas esta simples propriedade de linhas retas, que de certa forma e
evidente, quantos grandes problemas s ao esperados de um profundo
conhecimento da geometria ? Esta quest ao n ao pode ocorrer a uma
mente inquisidora da verdade; e fundamental determin a-la a n ao
perder tempo em adquirir o conhecimento
- Malton (sec. 18), New Royal Road to Geometry
2
o.
Metodo
Os Gregos conheciam um segundo metodo para o c alculo da circunferencia
da Terra utilizando uma estrela xa no ceu, em vez do Sol. Este 2
o.
metodo e
atribudo a Posidonius (135 - 51 a.c.), tutor de Cicero.
Posidonius observou que quando a estrela Canopus encontra-se no horizonte
sul de Rhodes, ela vista de Alexandria, ao sul de Rhodes, encontra-se acima
do horizonte formando um angulo de

24
, como mostra a gura 4. Para aplicar
este novo metodo, Posidonius teve que assumir as seguintes hip oteses;
12
unidade de dist ancia utilizada na Grecia antiga
Revista da ORM/SC n
o
3, 2006
Se a Terra n ao e Plana... 91
1. O arco, sobre a superfcie da Terra, ligando Rhodes e Alexandria encontra-
se sobre um grande crculo,
2. Os raios de luz vindos de Canopus chegam ` a Terra paralelos,

R
O
TERRA
CANOPUS

A
C

Figura 4: Metodo de Posidonius , =


Posidonius observou que o angulo central media

24
, o que implica, pelo
mesmo raciocnio de Erat ostenes, que conhecendo-se o comprimento do arco

AR de Alexandria a Rhodes, a circunferencia da Terra seria de C = 48.AR.


Entretanto, Alexandria est a separada de Rhodes pelo Mar Mediterr aneo,
donde havia grande diculdade em determinar o valor de

AR. Com enorme
inconsistencia, Posidonius utilizou o c alculo de Erat ostenes que havia estimado

AR = 3750 est adios. Consequentemente, pelo metodo de Posidonius, a circun-


ferencia da Terra mede C = 180.000 est adios; ou seja C = 28.350 km. Desta
forma, o erro obtido e da da ordem de 7%, muito maior do que o obtido por
Erat ostenes.
Todo este esfor co matem atico s o teria sentido se os viajantes e navegadores
acreditassem no fato de que a Terra e uma esfera. Sem duvida, ao sabermos o
valor do raio da terra, ca mais simples obtermos as dist ancias, pois o valor de
j a era conhecido com precis ao de 2 casas decimais.
Aplica cao
O valor de 180.000 est adios(28.350 km), divulgado pelo trabalho ampla-
mente conhecido do ge ografo Grego Strabo (64 a.c. - 23 d.c.), levou a con-
Revista da ORM/SC n
o
3, 2006
92 Artigo
sequencias mais profundas do que qualquer outro erro geometrico j a cometido.
Colombo usou a medida obtida por Posidonius para argumentar a viabilidade
de sua proposta de alcan car as Indias nagevando para o Oeste. Colombo exp os
sua proposta para s abios da epoca que acreditavam na hip otese do Mundo ser
redondo, e que eram encarregados para decidir se a viagem sobre o imenso
oceano em pequenos barcos de madeira tinha chance de sucesso. Eles n ao
estavam preocupados se os barcos poderiam cair em alguma especie de pen-
hasco no m do Mundo, eles de fato procupavam-se com a possibilidade do
apodrecimento dos barcos e do sacrifcio da tripula c ao que correria riscos de
morrer de fome e de sede. A decis ao dependia da estimativa para a prov avel
dist ancia que as naus navegariam, a qual dependia das estimativas feitas sobre
a circunferencia da Terra.
Colombo defendeu sua tese para os conselheiros do Rei e Rainha de Espanha
citando o grande astr onomo e geogr afo Ptolomeu que viveu muitos anos ap os
Erat ostenes, Posidonius e Strabo. De acordo com Ptolomeu, um viajante que
come casse sua viagem do ponto mais a Oeste no continente europeu, situado no
Cabo de S ao Vicente em Portugal, e rumasse Leste, sobre um mesmo paralelo,
ate retornar ao ponto de partida, faria a primeira parte da viagem por terra
e a segunda por mar. Isto signicaria que as terras dos continentes europeu e
asi atico ocupavam cerca de 180
o
de um paralelo no hemisferio norte.
C V
Figura 5
Na gura 5 temos que O e o
centro da Terra enquanto sobre o
mesmo paralelo temos os pontos V
e C correspondendo ao cabo de S ao
Vicente e ao extremo leste na China,
respectivamente(

V C = 180
o
). De
acordo com a teoria de Ptolomeu,
havia muita agua entre a China e
Portugal, o que n ao serviria para os
argumentos de Colombo. Assim, ele
fez uso da estimativa puramente es-
peculativa de um astr onomo grego
chamado Marinus de Tyre, a quem
Ptolomeu havia citado apenas para critic a-lo. Seguindo Marinus, Colombo
sup os que a dist ancia entre os pontos V e C, por terra, era de 225
o
. Portanto,
o ponto C foi deslocado para C
1
na gura 6.
Revista da ORM/SC n
o
3, 2006
Se a Terra n ao e Plana... 93
Observando um atlas moderno, observa-se que a dist ancia, em graus, dos
extremos dos continentes sobre o paralelo em que encontra-se V, e da ordem
de 120
o
; o que implica que a dist ancia por mar e da ordem de 240
o
. De ac ordo
com a estimativa de Colombo, a dist ancia por mar seria de 360
o
225
o
= 135
0
.
C V
C1
Figura 6
Utilizar a estimativa de Mari-
nus foi um dos muitos artifcios
que Colombo usou para diminuir a
dist ancia por mar ` as

Indias. Gra cas
a inven c ao da imprensa, por volta de
1450, pelo alem ao Johannes Guten-
berg, o livro do mercador veneziano
Marco Polo, descrevendo as suas via-
gens por terra ao oriente, havia sido
publicado. Colombo havia adquirido
uma c opia do livro de Marco Polo;
este exemplar, sobre o qual Colombo
anotava, ainda existe. As anota c oes
mostram como ele foi diminuindo
a dist ancia para alcan car as

Indias
navegando para Oeste. De acordo com Marco Polo, com um pouco de exa-
gero, a dist ancia, em graus, de V ao ponto no extremo leste da China, sobre
o paralelo, localizava-se a 28
o
mais afastado do que a estimativa de Marinus,
deslocando o ponto C
1
para o ponto C
2
, como indica a gura 7. Desta forma,
a dist ancia por mar seria de fato de 360
o
(225
o
+ 28
o
) = 107
o
. Marco Polo
armou que e as

Indias cavam em algum lugar pr oximo a Cipango(Jap ao), o
qual Colombo estimou como 30
o
mais a leste da China.
De acordo com o relato de Marco Polo, Cipango deveria encontrar-se onde
est a marcada a letra J na gura 7. Consequentemente, a dist ancia por terra de
V a C seria de 225
o
+28
o
+30
o
= 283
o
, da onde conclui-se que o arco

JV tem
a dist ancia em graus dada por 360
o
283
o
= 77
o
; correspondendo a dist ancia
por mar do Cabo de S ao Vicente a Cipango. Espertamente, Colombo plane-
jou partir das Ilhas Can arias, as quais encontravam-se, segundo estimativas da
epoca, a aproximadamente 9
o
a oeste do cabo de S ao Vicente, o que impli-
caria que a dist ancia a ser navegada seria de uns 68
o
, desprezando-se o fato
das Ilhas Can arias n ao se encontrarem sobre o mesmo paralelo que o Cabo.
Aparentemente, Colombo n ao tinha a precis ao como uma das suas virtudes.
Revista da ORM/SC n
o
3, 2006
94 Artigo
C V
C1 C2 J
Figura 7
Insatisfeito com os 68
o
obtidos, ele resolveu cortar mais 8
o
do seu caminho.
Assim, Colombo anunciou ao comite, espantado pelos argumentos apresenta-
dos, que ele chegaria ` as Indias nagevando 60
o
a Oeste das Ilhas Can arias, o que
corresponderia a 1/3 da estimativa de Ptolomeu, que na epoca ainda gozava
de grande prestgio.
C V
C1 J C2
K
23
o
Figura 8: JK = 68
o
, K= Ilhas Can arias e = 23
o
Para discutir a viabilidade da viagem, era necess ario transformar a dist ancia
estimada em 60
o
graus para quil ometros. Aqui o erro de Posidonius cumpriu a
sua fun c ao dentro dos objetivos de Colombo. Considerando que a circunferencia
da Terra calculada por Posidonius era de 180.000 est adios, os 60
o
nagevados
sobre o equador correspondem a
d =
60
360
180.000 = 30.000 est adios,
ou, em km, d = 4.725km.
Revista da ORM/SC n
o
3, 2006
Se a Terra n ao e Plana... 95
No entanto, Colombo n ao pretendia navegar ao longo do Equador, mas ao
longo de um paralelo de latitude, o que resultaria numa dist ancia ainda menor.
Para calcular o raio de um crculo de latitude e necess ario saber que a latitude
das Ilhas Can arias e de 23
o
. Isto signicaria que a dist ancia deveria ser da
ordem de 4.320 km.
Colombo concluiu sua exposi c ao ao comite dizendo que O nal da Espanha
e o come co das Indias n ao encontram-se muito distantes, o mar que os separa
e naveg avel em poucos dias tendo ventos favor aveis; ele estimou a viagem em
30 dias. Estes dizeres est ao gravados em uma das anota c oes feitas nas margens
de um livro seu sobre cosmograa.
Tendo Colombo como Almirante, a esquadra formada pelas caravelas Santa
Maria, Ni na e Pinta, partiu das Ilhas Can arias em setembro de 1492 e, em 33
dias, no dia 12 de outubro de 1492, alcan cou terra a uma dist ancia de 57
o
a
Oeste do ponto de partida, conforme previsto por Colombo. Estas terras n ao
faziam parte do Jap ao, mas de um Mundo Novo.
Rela c oes Metricas Esfericas
Distancia sobre a Esfera
Uma vez que a Terra n ao e plana, os axiomas da geometria euclideana e as
suas consequencias n ao podem ser empregadas para obtermos rela c oes metricas
entre comprimentos e angulos sobre a esfera. A diferen ca entre as geometrias
ca evidente quando observarmos que n ao h a retas sobre uma esfera.
No que segue, vamos assumir que a Terra e uma esfera; de fato, a Terra e
achatada nos p olos. Sendo assim, vamos abstrair o problema para a superfcie
de uma esfera com raio R.
Uma esfera de raio R centrada na origem e o conjunto dos pontos
(R) = (x, y, z) R
3
[ x
2
+y
2
+z
2
= R
2
.
Para melhor descrevermos os pontos sobre a esfera (R) introduzimos co-
ordenadas esfericas. Um sistema de coordenadas esfericas sobre e um par
(U, ) tal que U e um subconjunto aberto de e : (0, 2) (0, ) U
e um difeomorsmo
13
.
13
fun c ao diferenci avel que e uma bije c ao e cuja inversa tambem e diferenci avel
Revista da ORM/SC n
o
3, 2006
96 Artigo
Exemplo: Sejam N = (0, 0, 1) e S = (0, 0, 1) os p olos norte e sul e l =
(x, 0, z) [ x
2
+ y
2
= 1, x 0. Considere U = N, S l e : (0, 2)
(0, ) U denida por
(, ) = R.(cos()sen(), sen()sen(), cos()). (4)
Neste caso, temos que (U, ) e um sistema de coordenadas sobre e, se
p = (, ), as coordenadas esfericas de p = (x, y, z) (R) s ao (, ) (g. 9).
Os pontos p, q (R) denem o plano
pq
, que contem a origem e e gerado
pelos vetores op e oq;

pq
= s. op +t. oq [ s, t R.
A interse c ao de (R) com
pq
e uma circunferencia que denominamos de
equador e denotamos por e
pq
. Alem disto, os pontos p e q dividem o equador
e
pq
em dois arcos e
1
pq
e e
2
pq
denominados segmentos.
Para obtermos rela c oes metricas sobre (R) assumiremos os seguintes axi-
omas;
Axioma 1 : Dados dois pontos p, q (R) existe um unico equador e
pq

(R) tal que p, q e
pq
.
Axioma 2 : Os pontos p e q dividem o equador e
pq
em dois segmentos e
1
pq
e
e
2
pq
.
Axioma 3 : A dist ancia esferica entre dois pontos distintos p, q (R) e
d
(R)
(p, q) = inf(L(e
1
pq
), L(e
2
pq
)).
Axioma 4 : Para qualquer par de equadores e
1
e e
2
h a uma transforma c ao
f : (R) (R) que preserva as dist ancias entre pontos de (R) e f(e
1
) = e
2
.
Ao supormos que o angulo entre os vetores op e oq mede , a dist ancia
entre p e q (g.10) e dada por
d
S
2
(R)
(p, q) = R. = R.arcos
_
< op, oq >
R
2
_
(5)
Revista da ORM/SC n
o
3, 2006
Se a Terra n ao e Plana... 97
x
y
z
p

Figura 9: coordenadas esfericas


O
q

p
Figura 10: dist ancia esferica
Vamos considerar que, em coordenadas esfericas, os pontos p, q (R) s ao
descritos por
p = R.(cos(
p
)sen(
p
), sen(
p
)sen(
p
), cos(
p
)),
q = R.(cos(
q
)sen(
q
), sen(
q
)sen(
q
), cos(
q
)).
Ao substituirmos na express ao 5 obtemos a seguinte f ormula para a dist ancia:
d
S
2
(R)
(p, q) = R.arcos
_
cos().cos() + 2sen
2
_

2
_
.cos(
p
).cos(
q
)
_
,
(6)
onde =
q

p
e =
q

p
.
Se supormos que p pertence ao plano-xy, isto e,
p
= /2, segue que
d
S
2
(R)
(p, q) = R.arcos (cos().cos()) . (7)
De acordo com o axioma 4, existe uma transforma c ao em (R) preservando
a dist ancia e levando p ao ponto (1, 0, 0). Assim, podemos assumir que p
pertence ao plano-xy e
p
= /2. Portanto, a dist ancia entre os pontos p e q
sobre (R) satisfaz a identidade
cos
_
d
(R)
(p, q)
R
_
= cos().cos(). (8)
Revista da ORM/SC n
o
3, 2006
98 Artigo

Figura 11: angulo entre segmentos


Deni cao: O angulo formado por dois segmentos e igual ao angulo formado
pelos planos que contem os segmentos (g. 13).
Triangulos Esfericos
Os pontos A, B e C sobre (R), quando n ao pertencem a um mesmo
equador, denem um tri angulo ABC esferico. Assim como na geometria
euclideana, na geometria esferica existem rela c oes entre as medidas dos lados
com as medidas dos angulos de um tri angulo.
Teorema 1 : Teorema de Pit agoras Esferico - Seja ABC um tri angulo
geodesico sobre a esfera (R) tal que no vertice A o angulo seja ret angulo.
Suponha que a hipotenusa mede a, o lado oposto ` a B mede b e c seja a medida
do lado oposto ` a C. Ent ao,
cos
_
a
R
_
= cos
_
b
R
_
.cos
_
c
R
_
. (9)
Demonstra cao: De acordo com o Axioma 4, podemos considerar o lado AB
sobre o equador = /2. Em particular, podemos assumir que
Revista da ORM/SC n
o
3, 2006
Se a Terra n ao e Plana... 99
A = (1, 0, 0), B = (cos
_
c
R
_
, sen
_
c
R
_
, 0) e
C = (sen
_

2

b
R
_
, 0, cos
_

2

b
R
_
).
Portanto,<

OB,

OC >= cos
_
a
R
_
= cos
_
b
R
_
.cos
_
c
R
_
.
B
C
b
A c
a

Figura 12
Conforme dito anteriormente, a
nossa experiencia cotidiana mostra
que a express ao 1 e adequada para
resolvermos problemas de medi c ao
quando, por exemplo, queremos
medir as dimens oes de uma con-
stru c ao, ou de terrenos e ate as
dist ancias dentro de uma cidade.
Sendo assim, a identidade 1 deve
ser obtida a partir de 9 quando
os lados a, b e c do tri angulo s ao
muito pequenos em rela c ao ao raio R.
Por exemplo, para um segmento AB
medindo 10 metros sobre a superfcie da Terra temos que
10
6378, 11
1 mm.
Vejamos o que ocorre ao assumirmos na identidade (6) que
a
R
0,
b
R
0 e
c
R
0.
Segue das series de Taylor da fun c oes cosseno e seno que
a
R
0 cos(
a
R
) = 1
(
a
R
)
2
2
+o((
a
R
)
4
), (10)
sen(
a
R
) =
a
R
o((
a
R
)
2
). (11)
onde
Revista da ORM/SC n
o
3, 2006
100 Artigo
lim
a
R
0
o((
a
R
)
4
)
(
a
R
)
2
= 0.
As aproxima c oes 10 e 11, quando aplicadas ` a identidade 9, resultam em
1
1
2
_
a
R
_
2
+o(
a
R
4
) =
_
1
1
2
_
b
R
_
2
+o(
b
4
R
4
)
_
.
_
1
1
2
_
c
R
_
2
+o(
c
4
R
4
)
_
(12)
Consequentemente,
a
2
b
2
c
2
2
=
1
4
b
2
.c
2
R
2
+
1
2R
2
_
b
2
.o(
c
4
R
4
) +c
2
.o(
b
4
R
4
)
_
+ (13)
+
_
o(
a
4
R
4
) o(
b
4
R
4
) o(
c
4
R
4
)
_
o(
b
4
R
4
).o(
c
4
R
4
) (14)
Assim, se R a, R b e R c, ent ao
a
2
b
2
+c
2
,
sendo que no limite
a
R
0,
b
R
0 e
c
R
0
a
2
= b
2
+c
2
.
Portanto, o Teorema de Pit agoras euclideano deve ser aplicado quando os
lados do tri angulo s ao muito pequenos em rela c ao ao raio R, embora ele s o vale
nas situa c oes limites descritas acima ou quando a, b e c s ao xos e R .
Agora, vamos considerar um tri angulo qualquer.
Proposi cao 1 : Lei dos Cossenos - Seja ABC um tri angulo esferico em
com angulos internos medindo , e e cujos lados opostos medem a, b e c,
respectivamente. Ent ao,
Revista da ORM/SC n
o
3, 2006
Se a Terra n ao e Plana... 101
cos() =
cos
_
a
R
_
cos
_
b
R
_
cos
_
c
R
_
sen
_
b
R
_
.sen
_
c
R
_
,
cos() =
cos
_
b
R
_
cos
_
a
R
_
cos
_
c
R
_
sen
_
a
R
_
.sen
_
c
R
_ ,
cos() =
cos
_
c
R
_
cos
_
a
R
_
cos
_
b
R
_
sen
_
a
R
_
.sen
_
b
R
_ .
(15)
Demonstra cao: Sem perda de generalidade, suponha que
A = (1, 0, 0), B = (cos(
B
)sen(
B
), sen(
B
)sen(
B
), cos(
B
))
C = (cos(
C
), sen(
C
), 0).
(16)
Assim,
cos(
a
R
) = <

OB,

OC >= cos(
C

B
)sen(
B
),
cos(
b
R
) = <

OA,

OC >= cos(
C
),
cos(
c
R
) = <

OA,

OB >= cos(
B
)sen(
B
);
(17)
da onde segue que,
sen(
a
R
) =
_
cos
2
(
B
) +sen
2
(
C

B
)sen
2
(
B
)
sen(
c
R
) =
_
cos
2
(
B
) +sen
2
(
B
)sen
2
(
B
).
(18)
Os vetores
Revista da ORM/SC n
o
3, 2006
102 Artigo
n
AB
=

OA

OB
[

OA

OB [
= (
(0, cos(
B
), sen(
B
)sen(
B
))
_
cos
2
(
B
) +sen
2
(
B
)sen
2
(
B
)
n
BC
=

OB

OC
[

OB

OC [
=
(cos(
B
)sen(
C
), cos(
B
)cos(
C
), sen(
C

B
)sen(
B
))
_
cos
2
(
B
) +sen
2
(
C

B
)sen
2
(
B
)
n
CA
=

OC

OA
[

OC

OA [
= (0, 0, 1)
(19)
determinam os planos
AC
,
AB
e
BC
, respectivamente. Considere
n
AB
, n
BC
, n
CA
uma base orientada de R
3
. Uma vez que,
cos() = < n
AC
, n
AB
>,
cos() = < n
AB
, n
BC
>,
cos() = < n
AC
, n
BC
>,
obtemos
cos() =
sen(
B
)sen(
B
)
_
cos
2
(
B
) +sen
2
(
B
)sen
2
(
B
)
cos() =
cos
2
(
B
)cos(
C
) sen(
C

B
)sen
2
(
B
)sen(
B
)
_
cos
2
(
B
) +sen
2
(
B
)sen
2
(
B
)
_
cos
2
(
B
) +sen
2
(
C

B
)sen
2
(
B
)
cos() =
sen(
C

B
)sen(
B
)
_
cos
2
(
B
) +sen
2
(
C

B
)sen
2
(
B
)
.
(20)
Da rela c ao 16, temos
cos
_
a
R
_
=cos
_
b
R
_
cos
_
c
R
_
+sen
_
b
R
_
sen(
B
)sen(
B
)
sen(
C

B
)sen(
B
) =sen
_
b
R
_
cos
_
c
R
_
cos
_
b
R
_
sen(
B
)sen(
B
),
(21)
Revista da ORM/SC n
o
3, 2006
Se a Terra n ao e Plana... 103
e, consequentemente,
sen(
B
)sen(
B
) =
cos
_
a
R
_
cos
_
b
R
_
cos
_
c
R
_
sen
_
b
R
_
sen(
C

B
)sen(
B
) =
cos
_
c
R
_
cos
_
a
R
_
cos
_
b
R
_
sen
_
b
R
_ .
(22)
As express oes 21 aplicadas ` a 16 resultam nas seguintes identidades:
cos() =
cos
_
a
R
_
cos
_
b
R
_
cos
_
c
R
_
sen
_
b
R
_
.sen
_
c
R
_
,
cos() =
cos
_
c
R
_
cos
_
a
R
_
cos
_
b
R
_
sen
_
a
R
_
.sen
_
b
R
_
.
(23)
Analogamente, a identidade para o cos() e obtida a partir da situa c ao na
qual os vertices do ABC s ao
A = (1, 0, 0), B = (cos(
B
), sen(
B
), 0)
C = (cos(
C
)sen(
C
), sen(
C
)sen(
C
), cos(
C
)).
(24)
Neste caso, obtemos
cos() =
cos
_
b
R
_
cos
_
a
R
_
cos
_
c
R
_
sen
_
a
R
_
.sen
_
c
R
_
.
Revista da ORM/SC n
o
3, 2006
104 Artigo
Corolario 1 : Lei dos Senos - Num tri angulo esferico ABC, como na
proposi c ao 1, valem as identidades
sen()
sen(a)
=
sen()
sen(b)
=
sen()
sen(c)
. (25)
A seguir, como no caso do Teorema de Pit agoras, vamos analisar as ex-
press oes 14 quando R a, R b e R c. Ao aplicarmos as aproxima c oes
10 e 11 ` a primeira express ao em 14, segue que:
cos() =
_
1
1
2
a
2
R
2
+o(
a
4
R
4
)
_

_
1
1
2
b
2
R
2
+o(
c
4
R
4
)
_
.
_
1
1
2
c
2
R
2
+o(
1
R
4
)
_
_
b
R
o(
b
4
R
2
)
_
.
_
c
R
o(
c
4
R
2
)
_ =
=
b
2
+c
2
a
2
2

1
4
b
2
c
2
R
2
bc
1
R
3
.
_
b.o(
c
4
R
4
) +c.o(
b
4
R
4
)
_
+
1
R
2
o(
b
4
R
4
).o(
c
4
R
4
)
+
+
R
2
.[o(
a
4
R
4
) o(
b
4
R
4
) o(
c
4
R
4
)]
bc
1
R
3
.
_
b.o(
c
4
R
4
) +c.o(
b
4
R
4
)
_
+
1
R
2
o(
b
4
R
4
).o(
c
4
R
4
)
+
+
1
2
[b
2
.o(
c
4
R
4
) +c
2
.o(
b
4
R
4
)]
1
R
2
.o(
b
4
R
4
).o(
c
4
R
4
)
bc
1
R
3
.
_
b.o(
c
4
R
4
) +c.o(
b
4
R
4
)
_
+
1
R
2
o(
b
4
R
4
).o(
c
4
R
4
)
.
Portanto, no limite
a
R
0,
b
R
0 e
c
R
0 vericamos a identidade
a
2
= b
2
+c
2
2bc.cos() (26)
A

Area de um Triangulo Esferico
Uma vez que as rela c oes metricas em tri angulos esfericos s ao simples, e
natural que haja uma express ao para a area.
Revista da ORM/SC n
o
3, 2006
Se a Terra n ao e Plana... 105
Um gomo em e uma regi ao limitada por dois segmentos e ligando
os pontos antpodas p = (x, y, z) e q = (x, y, z), em . Em cada um dos
vertices p e q, os segmentos formam um angulo denominado o angulo do
gomo. Um gomo com angulo e equivalente, pelo axioma 4, ` a
G

= (cos()sen(), sen()sen(), cos()) [ 0 , 0 .


Lema 1 : A area de um gomo com angulo interno , em (R), e igual a 2R
2
.
Demonstra cao: Utilizando coordenadas esferica temos que
A = R
2
.
_

0
_

0
sen()dd = 2R
2
.
Em particular, se = 2 o resultado 4R
2
d a a area da esfera.
Teorema 2 : A area de um tri angulo esferico ABC (R), cujos angulos
internos medem , e , e
A = R
2
. [( + +) ] .
Figura 13
Revista da ORM/SC n
o
3, 2006
106 Artigo
Demonstra cao: Seja A a area do tri angulo, pelo lema anterior a area do gomo
G

com angulo e
A+A

= 2R
2
,
onde A

e a area da regi ao complementar ao tri angulo no gomo. Uma vez que


a area de e 4R
2
e que G

e um hemisferio, segue que


A+A

+A

+A

= 2R
2
Consequentemente,A+ (2R
2
A) + (2R
2
A) + (2R
2
A) = 2R
2
,
e A = R
2
. [( + +) ] .
REFER

ENCIAS
[1] C.M.Doria, Geometria sobre as Superfcies, notas.
[2] J.L.Heilbron, Geometry Civilized, Oxford, 1998.
[3] Enciclopedia Brit anica, 1995.
[4] Enciclopedia Larrousse Cultural, 1998.
[5] Eugenio Gomes, VIEIRA - Serm oes, 3
a
edi c ao, Ed. Agir, 1963.
Revista da ORM/SC n
o
3, 2006
Solues dos
Problemas
Propostos
108 Solu c oes de problemas propostos na revista anterior
Revista da ORM/SC n
o
3, 2006
109
1. Proposto por Ivan Pontual Costa e Silva, UFSC. Seja L uma reta, P
1
e P
2
pontos distintos fora de L mas coplanares com L. Mostre que existe um
unico ponto P de L com a propriedade que d(P, P
1
) +d(P, P
2
) e mnima,
e neste caso, o menor angulo que PP
1
faz com L e o menor angulo que
PP
2
faz com L s ao iguais.
SOLUC

AO(enviada pelo proponente)
Temos duas situa c oes possveis:
(i) P
1
e P
2
est ao em lados opostos com respeito a l;
(ii) P
1
e P
2
est ao do mesmo lado com respeito a l.
Caso (i):
P
P
P
l
P
Seja P
1
P
2
o segmento que une P
1
a P
2
. Estando P
1
e P
2
de lados opostos
de l, h a um ponto P de l entre P
1
e P
2
. Nesse caso,
d(P
1
, P
2
) = d(P
1
, P) +d(P
1
, P
2
) (1)
Seja P

um ponto qualquer de l, P

,= P. P

n ao pode ser colinear com


P
1
e P
2
, uma vez que a reta P
1
P
2
n ao pode intersectar l mais de uma
vez, e j a o fez em P.
Considere o tri angulo com vertices P

, P
1
e P
2
. A desigualdade triangular
garante que:
d(P

, P
2
) +d(P

, P
1
) > d(P
1
, P
2
) (2)
Da equa c ao (1), temos portanto:
Revista da ORM/SC n
o
3, 2006
110 Solu c oes de problemas propostos na revista anterior
d(P, P
1
) +d(P, P
2
) < d(P

P
1
) +d(P

, P
2
), (3)
isto e, d(P, P
1
) +d(P, P
2
) e a menor possvel, e P e o ponto procurado.
Seja o menor angulo que PP
1
faz com l. Nesse caso, e menor ou
igual a um angulo reto. Um dos angulos que PP
2
faz com l e oposto pelo
vertice, e portanto, congruente a , sendo esse imediatamente o menor
angulo.
Caso (ii):
Seja l

a perpendicular a l baixada por P


2
, e seja A o pe dessa perpen-
dicular em l. Temos dois subcasos:
(ii)-(a): P
1
est a em l

(ii)-(b): P
1
n ao est a em l

Subcaso (ii)-(a):
A
P
l
l
P
A
1
2
Seja A ponto arbitr ario de l, com A

,= A. Nesse caso, A n ao est ao em


l

. Os tri angulos P
2
AA

e P
1
AA

s ao tri angulos s ao ret angulos e P


2
A

e P
1
A

s ao as respectivas hipotenusas.
Portanto:
d(P
1
A

) > d(P
1
A) (4)
d(P
2
A

) > d(P
2
A) (5)
(P
1
A

) +d(P
2
A

) > (P
1
A) +d(P
2
A) (6)
E A e o ponto procurado.
Revista da ORM/SC n
o
3, 2006
111
Por constru c ao, o angulo menor que P
1
A e P
2
A formam com l s ao con-
gruentes, sendo ambos retos.
Subcaso (ii)-(b):
Seja P

2
o ponto em l

simetrico a P
2
com respeito a l (existe e e unico
por transporte de segmentos).
A A P
P
P
P
P
2
2
1
l
P

2
est a do lado oposto ao de P
1
com respeito a l, e portanto, existe um
ponto P fora de l entre P
1
e P

2
. P ,= A. Pelo caso lado- angulo-lado,
temos:
P
2
PA P

2
PA (7),
de onde
d(P
2
, P) = d(P

2
, P) (8)
Seja P um ponto qualquer de l.Se P=A temos d(P

2
, P

) = d(P
2
, P

) (9)
por constru c ao. Se P

,= A, temos que P

2
P

A P
2
P

A (10) pelo
caso lado- angulo-lado.
Logo
d(P

2
, P

) = d(P
2
, P

) (11)
Revista da ORM/SC n
o
3, 2006
112 Solu c oes de problemas propostos na revista anterior
Pelo caso (i),
d(P
1
, P

) +d(P

2
, P

> d(P
1
, P) +d(P

2
, P) (12)
Portanto:
d(P
1
, P

) +d(P
2
, P) > d(P
1
, P) +d(P
2
, P) (13)
e P e o ponto procurado.
Finalmente, o angulo = P
2

PA e o menor angulo que P
2
P faz com l,
por ser menor que um angulo reto. Pela congruencia de tri angulos (7),
temos:
P

2

PA

P
2

PA (14)
Seja A um ponto de l de modo que P esteja entre A e A. O angulo
P
1

PA

e oposto pelo vertice a P

2

PA, e portanto congruente a este.
Logo, P
1

PA

2

PA, sendo este o menor angulo.
2. Proposto por Andrzej Solecki, UFSC. Seja f(x) = x
3
3x
2
+ax+1. Quais
as 3 razes reais desta fun c ao, sabendo que as mesmas est ao em PA? Qual
o valor de a?
SOLUC

AO (enviada pelo leitor Vilmar Minella Junior)
Represente as 3 razes em PA: xr, x, x+r. Utilizando-se a rela c ao de
Girard, temos: xr+x+x+r =
b
1
, onde b = 3 xr+x+x+r =
b 3x =
(3)
1
x = 1.
Substituindo-se o valor da inc ognita x na equa c ao, encontraremos o valor
de a: x
3
3x
2
+ax+1 = 0 1
3
3(1)
2
+a(1) +1 = 0 13+a+1 =
0 1 +a = 0 a = 1.
Para o polin omio car divisvel, faremos:x = 1 ou x1 = 0. Assim sendo,
baixaremos o grau do polin omio.
a) Pelo metodo da chave:
Revista da ORM/SC n
o
3, 2006
113
x
3
3x
2
+x + 1 [ x 1
x
3
+x
2
x
2
2x 1
2x
2
+x + 1
2x
2
2x
x + 1
x 1
00
b) Pelo dispositivo pr atico de Briot-Runi:
1 3 1 1
1 1 2 1 0
Desta maneira transformamos a equa c ao de terceiro grau em uma equa c ao
de segundo grau :x
2
2x 1 = 0.
Podemos resolver esta equa c ao pelo metodo de B askara , ent ao:
x =
b
+

2a
= b
2
4ac = 4 + 4 = 8
x
1
=
2 +

8
2
x
1
=
2 + 2

2
2
x
1
= 1 +

2
x
2
=
2

8
2
x
2
=
2 2

2
2
x
2
= 1

2
3. Proposto por Ant onio Vladimir Martins, UFSC, retirado do livro Tech-
niques of Problem Solving. Com um grande quadrado de chumbo, e
possvel achar o centro de uma pequena circunferencia . Mas se voce
tiver um grande tri angulo equil atero, como achar o centro desta mesma
circunferencia?
Revista da ORM/SC n
o
3, 2006
114 Solu c oes de problemas propostos na revista anterior
D
O
E
D E
A
B
C
r
SOLUC

AO(enviada pelo graduando Felipe Vieira)
Sobre uma circunferencia, a qual estamos interessados em encontrar seu
centro, colocamos o vertice A do tri angulo eq uil atero ABC. Da interse c ao
dos lados AB e AC com essa circunferencia resultam os pontos D e E.
Girando o tri angulo eq uil atero ABC em torno do vertice A, de modo que
o lado AB passe pelo ponto E, encontramos a reta r dada pelo ponto lado
AC do tri angulo em sua nova posi c ao.
Agora, colocamos uma das bases do tri angulo eq uil atero sobre a reta r e
deslizamos horizontalmente sobre ela ate um dos lados tocar o ponto E
(de maneira que um dos lados do tri angulo n ao coincida com o lado AC) e
obtemos o ponto D (dado pelo vertice do tri angulo ABC com a reta r).
Revista da ORM/SC n
o
3, 2006
115
Fa camos este mesmo procedimento, girando o tri angulo eq uil atero ABC
em torno do vertice A, de modo que o lado AC passe pelo ponto E e
assim encontramos o ponto E.
Tra cando os segmentos DD

e EE

utilizando-se um dos lados do tri angulo


eq uil atero obtemos a partir da interse c ao desses segmentos o centro O da
circunferencia.
Observando que EE

=DD

e a dist ancia EE

e igual a

3
2
lado do
tri angulo ABC. Prove!
Provamos que e possvel encontrar o centro da circunfencia utilizando-se
de um tri angulo eq uil atero. Ser a possvel encontrar o centro dessa mesma
circunferencia utilizando-se qualquer polgono regular?
Revista da ORM/SC n
o
3, 2006
116 Solu c oes de problemas propostos na revista anterior
Revista da ORM/SC n
o
3, 2006
Problemas
Propostos
118 Problemas propostos
Revista da ORM/SC n
o
3, 2006
119
Convidamos o leitor a enviar solu c oes dos problemas propostos e a sugerir
novos problemas para as pr oximas edi c oes.
1. Proposto por Cesar Raitz, UFSC. Dado um quadrado de centro em O e
cuja medida dos seus lados e a. Um outro quadrado tem a medida dos
seus lados b, com b maior que a, e tal que um dos seus vertices e xo em
O e gira em torno de O. Qual deve ser a posi c ao do quadrado maior para
que o permetro da parte comum dos dois quadrados seja mnimo?
2. Proposto por Jucavo Savie Rocha, mestrando UFSC. Sejam 16 cartas de
baralho, do valete ao as ( J,Q,K,A) e dos quatro naipes (Ouro, Espada,
Copas,Paus), todas distintas. Disponha essas 16 cartas em um quadrado
quatro por quatro de forma que em nenhuma linha, coluna ou diagonal
se repitam duas cartas de mesma letra ou mesmo naipe.
3. Proposto por Cesar Raitz, UFSC. Os lados de um tri angulo medem re-
spectivamente 6 cm, 8 cm e 10 cm. Um disco de raio 1 cm rola no interior
do tri angulo sempre tangente em pelo menos um dos lados do tri angulo.
No momento em que o centro dos disco chega na posi c ao inicial de partida,
depois de ter feito uma volta completa no tri angulo, qual e a dist ancia
que ele percorreu?
4. Proposto por Lucas Spillere Barchinski, graduando UFSC. Qual a proba-
bilidade de amigo oculto com n pessoas de certo ( ou seja, ninguem pega
a si mesmo)?
5. Proposto por Fabiano Carlos Cidral, graduando UFSC. Prove que todo
subconjuto de 1003 elementos do conjunto A = 1, 2, 3, ..., 2004 possui
pelo menos dois elementos cuja soma e igual a 2005.
Revista da ORM/SC n
o
3, 2006
120 Problemas propostos
Revista da ORM/SC n
o
3, 2006
Outras
Olimpadas
122 Outras olimpadas
Revista da ORM/SC n
o
3, 2006
123
Resultados de alunos de SC em outras Olimpadas
Resultados na OBM
2004
Nvel 01
Renan Henrique Finder (Joinville) - Medalha de Prata
Vitor Costa Fabris (Crici uma) - Men c ao Honrosa
Nvel Universitario
Giuliano Boava (UFSC - Florian opolis) - Men c ao Honrosa
Revista da ORM/SC n
o
3, 2006
124 Outras olimpadas
Revista da ORM/SC n
o
3, 2006
Informaes Gerais
126 Informa c oes gerais
Revista da ORM/SC n
o
3, 2006
Envio de Problemas e Solu c oes 127
Envio de Problemas e Solu c oes
A se c ao de problemas propostos e solu c oes e uma se c ao din amica. Con-
tribua propondo problemas e enviando-nos suas solu c oes de qualquer prob-
lema proposto. Os problemas n ao devem exigir, de preferencia, conte udos de
matem atica de nvel universit ario, porem podem ter solu c oes alternativas us-
ando estes conte udos.
Envio de Artigos
Professores do ensino fundamental e medio, professores universit arios, bem
como alunos de gradua c ao e p os-gradua c ao est ao convidados a enviar seus
artigos para a revista.
Artigos submetidos para publica c ao ser ao analisados pela comiss ao edi-
torial. Os artigos devem abordar os temas de forma clara e n ao eminente-
mente tecnica e n ao devem necessitar, como pre-requisitos, conhecimentos de
matem atica de nvel universit ario.
N ao h a exigencia de um editor de texto em particular mas, caso o autor
conhe ca e utilize o L
A
T
E
X, ent ao o artigo poder a ser submetido neste formato.
Cadastramento
Diretores, coordenadores e professores de matem atica que desejarem que
seus alunos participem das olimpadas (OBM e ORM) podem cadastrar suas
escolas entrando no nosso site ou entrando em contato diretamente conosco
(ver abaixo).
Alunos interessados em participar das olimpadas de matem atica podem
consultar nosso site para vericar se a sua escola est a cadastrada. Caso contr ario,
devem solicitar a seus professores de matem atica que cadastrem a escola. Lem-
bramos que as olimpadas de matem atica s ao feitas para os alunos, n ao sendo
uma competi c ao entre escolas. Assim sendo, espera-se que as escolas estimulem
seus alunos a participar e que, no mnimo, apoiem aqueles alunos que assim o
desejarem.
Como adquirir a revista
Esta revista est a sendo distribuda gratuitamente a diversas escolas do es-
tado de Santa Catarina (um exemplar por escola). Escolas que n ao receberam
Revista da ORM/SC n
o
3, 2006
128 Informa c oes gerais
a revista podem nos solicitar o envio da mesma.
Erramos
Na Revista n
o
2, tem-se as seguintes altera c oes:
i) Na p agina 103, onde se le (a b)
2
0, deve-se ler (a b)
2
0;
ii) Na p agina 116, onde se le F24 = 2
24
+ 1, deve-se ler F
24
= 2
2
24
+ 1.
Fale Conosco
Entre em contato conosco para esclarecer sua d uvidas, dar sugest oes ou
fazer corre c oes por:
Nosso site: www.orm.mtm.ufsc.br
Telefone/Fax: (48) 3316809 (PET - Matem atica)
e-mail: orm@pet.mtm.ufsc.br
Endere co: PET - Matem atica
Departamento de Matem atica - CFM
UFSC
Campus Universit ario - Trindade
88040-900 Florian opolis/SC
Revista da ORM/SC n
o
3, 2006

Вам также может понравиться